You are on page 1of 54

* NLE * NCLEX * CGFNS * HAAD * PROMETRICS * DHA * MIDWIFERY * LET * RAD TECH * CRIMINOLOGY * DENTISTRY * PHARMACY *

RECALLS 1 EXAM
July 2021 Philippine Nurse Licensure Examination Review

NAME: ZAMORA, MARIA KARINA D.

A. Any preliminary observations – not


any, put what is only actually related
Nursing Practice I
B. Preliminary observations
Situation 1- Quality documentation and reporting C. Patient‟s response
are necessary to enhance efficient, D. Nursing measure taken for negative
individualization patient care. Nurse Gigi is response
assigned to Maria who is diagnosed with diabetes.
4. Nurse Gigi is making a discharge plan. The items
1. Nurse Gigi is aware that a record should contain she needs to document and report should
descriptive, objective observations about what include, EXCEPT:
the nurse sees, hears, feels and smells. The A. Needs for referrals
following describes what a record ought to be: B. Nurses‟ observations
A. Maria is uncooperative as manifested by C. Client‟s involvement in the care plan
her refusal to take a bath – “is” – D. Patient‟s goals or expected outcomes
diniktahan ; should be “appears” – this is in nsg process, discharge plan is
B. Maria appears depressed – “is” – for home care
diniktahan ; should be “appears” 5. Nurses are ethically obligated to information
C. Maria is anxious as manifested by her about client‟s illness and treatment confidential.
tossing in bed and disarrayed beddings The person who has the legitimate access to the
D. Patient states, “I wish to end my life” patient‟s record is the .
– as verbalized, walang halong judgment A. Husband
2. When documenting the intake-output of Maria, C. attending doctor – found in magna carta
Nurse Gigi should remember that she must use of pt’s rights and obligations
precise measurements to ensure accuracy. B. Primary nurse
Which of the following reflects accuracy? D. patient
1. Maria drank an adequate amount of fluid Situation 2- Nurse Ben is assigned in the
during the shift. outpatient department. He admits Susing, 25
2. Maria‟s intake during the shift is 360 ml. years old, who is complaining of moderate to
3. Maria‟s wound is 3 cm in length. severe abdominal pain over the right iliac region
4. Maria‟s wound is large and gaping. for the last 2 days. This is accompanied by
frequent urination.
A. 1 and 3
6. The FIRST STEP that nurse Ben should do is to
B. 2 and 3 – should be specific
A. Percuss the abdomen
or has measurement
B. Get the vital signs
C. 3 and 4
C. Palpate the right iliac region
D. 1 and 4 D. Interview Susing
3. Another guideline for good documentation and 7. An important added data that Nurse Ben should
reporting that Nurse Gigi recalls is about have if he is thinking of a urinary problem is
completion. When nurse Gigi administers
A. Presence of blood in the urine
medications, the following should be included,
B. Presence of pain on urination
EXCEPT:

TOPRANK REVIEW ACADEMY INC.


RECALLS 1 EXAM Page 1
C. When the pain appears whether at the start B. Philippine Nurses Association
or end of urination C. Professional Regulation Commission
D. Frequency, amount and burning sensation D. Commission on Higher Education
on urination 13. What Nursing Republic Act should Nurse Leah
8. An important added data that nurse ben should refer too?
ask Susing, should we consider the presence of A. RA 9371 C. RA 7664
a gynecological problem is . B. RA 7164 – old law D. RA 9173
A. Date of menarche – if risk for Cervical CA 14. Since she needs to work in a hospital to get
B. Date of last menstrual period hospital experience, which part of the Nursing
C. Her age and civil status – not specific to Law should Leah focus for her safe practice as a
gynecology novice nurse?
D. Occupation – not specific to gynecology A. Scope of Nursing Practice – prevent
9. An important added data that Nurse Ben should malpractice
ask Susing, should we consider the existence of B. Nursing Research
a surgical problem is . C. Nursing Education
A. Fever – infection C. constipation D. Nursing Personal System
B. Vomiting D. character of pain 15. When Leah eventually gets employed in the
10. There are two important laboratory examinations hospital, she will initially be required to utilize
that could validate Nurse Ben‟s presumptions on which of the following in rendering care to her
the case which are . clients?
A. CBC and stool examination A. Traditional and innovative approaches
B. Urinalysis and CBC – indicates bleeding B. Therapeutic use of self
C. Stool and urine examinations C. Health teachings
D. Urinalysis and vaginal examinations D. Nursing process

Situation 3- Nurse Leah has just passed the Situation 4- Alex, 15 years old, has been
Nursing Licensure Examination. She needs to complaining of sleep disturbance for the past 3
refresh herself regarding the law that governs the weeks. He related that he feels weak and has no
practice of nursing in the Philippines. vigor in doing his school work. He is brought to
hospital B for specialization.
11. By virtue of section 16 of the Philippine Nursing
Law, the first step she needs to do after passing 16. As his attending nurse, what will be the best
the Nursing Licensure Examination is to . question to ask in order to find out Alex‟s cause
A. Take the oath of the profession before of sleep disturbance?
the Professional Regulatory Board of A. “Could you tell me what you feel about
Nursing – no license if no oath taking school work?”
B. Apply for the Certificate of B. “Is there something or anyone who is
Registration/Professional License\ bothering you in school?”
C. Get an official copy of her board rating C. “Could you share with me any major
from the PRC office problems that you have encountered in
D. Apply for a Professional Identification Card school?”
12. Which government body has the power to D. “What is bothering you in school?” –
revoke or suspend Leah‟s certificate of broad question, para makuha yung concern
registration/professional license, should she niya
commit unprofessional and unethical conduct in 17. The nursing diagnosis appropriate for Alex is
the future? A. Insomnia
A. Professional Regulatory Board of B. Sleep deprivation related to problems in
Nursing – Quasi Judicial Power ; revoke: school
confiscated ; suspended: hindi lang muna C. Disturbed sleep pattern related to
pwede mag-work pero na sayo pa rin license school problem
mo D. Readiness for enhanced sleep
TOPRANK REVIEW ACADEMY INC.
RECALLS 1 EXAM Page 2
18. In order to help Alex go to sleep, what simple 6. Cover with thick blankets to
and inexpensive nursing intervention can the stimulate sweating
nurse provide considering the type of room Alex A. 1, 3, 5
is confined in? B. 1, 5, 6
A. Room should be well ventilated C. 2, 4, 6
B. Light should be dimmed at specific D. 2, 3, 4
time – part of sleep hygiene – things you 23. Client Jimmy wishes to know his present
have to do to have better sleep temperature in Farenheit. Nurse Yana‟s answer
C. Putting on the air conditioning unit is which of the following?
D. Putting on dark-colored curtains A. 102.2F
19. If Alex cannot sleep immediately, which of the B. 101.3F
following can be provided by the nurse? A C. 100.4F
A. glass of warm milk D. 99.5F : Celsius x 9 / 5 + 32
B. A bedside story 24. Client Jimmy complains of severe chilly
C. a sleeping pill sensation. What regulatory function of the
D. a warm bed bath hypothalamus describes his reaction?
20. In terms of nursing care, the BEST help that the A. heat production - thermogenesis
nurse can do is to B. heat promotion
A. Avoid giving round the clock medications C. heat adaptation
B. Refrain from clutter at the nurse‟s station D. heat conversation
C. Omit vital sign tasking during the sleeping 25. After client Jimmy‟s body temperature stabilized
time of Alex to normal, he began to complain of feeling hot
D. Tone down nurse’s voice and finds difficulty of adjusting to the climate
two days after. What is the APPROPRIATE
Situation 5- Client Jimmy, 66 years old, a answer to nurse Yana?
foreigner, was admitted to the hospital due to A. “Foreigners cannot adjust to hot
high fever for five days. His temperature on temperatures”
admission was 37.5 „C with flushed and warm to B. “People from cold regions have less brown
touch skin. Nurse Jana, who is his attending adipocytes”
nurse, gave him a cool sponge bath. C. “Those who lived in cold places have lower
metabolic rate than those who live in tropical
21. Cool sponge bath is an independent intervention region”
of nurse Jana aimed at D. “People from cold regions have more
A. Alleviating discomforts of client Jimmy brown adipocytes”
B. Generalizing full sponge bath care to client 26. Nurse Susan develops patient safety goals. What
Jimmy get hospital admission policy nursing goal is APPROPRIATE for safety
C. Diverting the temperature of client environment? Select all that apply.
Jimmy – conduction (solid to solid) – from 1. Reduce the risk for patient harm
person to sponge resulting from falls
D. Normalizing the vital signs of client Jimmy 2. Create a clean, orderly environment
22. The independent nursing intervention to reduce of patient
temperature of client Jimmy include which of the 3. Prevent high concentration drug
following? Select all that apply. errors
1. Monitor temperature regularly 4. Develop clean and simple signage for
2. Offer only cold drinking water directions
frequently A. 1, 2 & 3
3. Provide tepid sponge bath B. 1, 2 & 4
4. Administer Paracetamol every 4 hours C. 2, 3 & 4
5. Encourage wearing loose cotton D. 3 & 4
clothing

TOPRANK REVIEW ACADEMY INC.


RECALLS 1 EXAM Page 3
27. Nurse Susan should prevent or reduce infectious A. Put him near the electric fan to assist him in
agents PRIMARILY through the following breathing
practices. Select all that apply. B. Ask him what triggered his attack this time
1. Clean the room with antiseptic C. Place him in a comfortable
solutions – antiseptic is for living, environment
disinfectant for non-living D. Put him near the nurse‟s station so that he
2. Proper garbage disposal could be assessed often
3. Mandatory hand washing upon entry 32. What particular important vital sign should Nurse
to hospital Chard monitor and report to the physician?
4. Improve safety administration of A. Respiratory rate
medicines B. blood pressure
A. 1, 2 & 3 C. Cardiac rate
B. 2, 3 & 4 D. Pulse rate
C. 1 & 3 33. Client Alan knows that he will be given
D. 1 & 4 decongestant through inhalation. What will be
28. Nurse Susan must comply with the policies of the instruction of Nurse Chard? That he should
health care facilities regarding isolation breathe and that the noozle of the NEBULIZER
precautions. The BASIC precaution in the should be MAINLY near his _____________.
hospital is _______________________. A. Nose only
A. posting of signs of silence B. Face
B. limited visitors who are within the patient’s C. Mouth only
age D. Nose and mouth
C. private room placement 34. Respiratory function is altered in clients with
D. strict utilization of personal things asthmatic attack. Which of the following is the
29. Nurse Susan includes air-borne precautions in cause of this alteration that Nurse Chard should
addition to Standard Precautions to be observed understand in order for him to make a good
at all times. These include, EXCEPT _______. nursing care plan?
A. positioning and draping of patients A. Narrowing of the upper air passages –
B. hand washing techniques cause of asthma is bronchospasm or
C. wearing of masks and gloves bronchoconstriction
D. limited patient transport B. Increased airway resistance
30. To protect spreading infection from one patient C. Inadequate surfactant reaction
to another, Nurse Susan must not forget to wash D. Paradoxical movements of the chest wall
her hands _________. 35. An important health teaching that Nurse Chard
A. before and after making rounds and must give before client Alan’s discharge is
endorsement of all clients ________.
B. before and after performing procedures A. To avoid “sando” and shorts, if the weather
and in between patient care is cold
C. twice before and after all the procedures are B. to use attire he is presently wearing only
done to all the clients during summer time
D. apply alcohol before and after performing C. to watch out for climate change and
procedures to all patients unnecessary exposure to elements of
the environment that trigger his
Situation 6 - Client Alan, 33 years old, is known asthma attacks – avoid triggers
for his asthma attacks. His asthma attacks D. to be careful always because constant
commonly occur at the change of climate in his asthma attacks can trigger complication and
hometown. He was just wearing a “sando” death
{sleeveless shirt} and short pants when he was
admitted to the hospital. Situation 7 - Andrea underwent mastectomy of
her right breast. After surgery, she was instructed
31. What will be Nurse Chard‟s PRIORITY to have range of motion exercises
INTERVENTION?
TOPRANK REVIEW ACADEMY INC.
RECALLS 1 EXAM Page 4
36. What is the main purpose of this type of exercise B. health care providers
for Andrea? It is to ________________. C. zumba
A. prevent pain on the site of operation D. wellness
B. prevent contracture deformity of the 42. Which is the dependent variable?
muscle of the right arm and chest A. health provider
C. improve general circulation B. wellness
D. improve breathing C. exercise
*if elevate affected arm: prevent lymphedema D. effects
37. The isotonic exercise such as Range of Motion 43. The research design of the study is
exercises are those that are initiated by the ____________
________. A. quasi-experimental
A. patient B. experimental – if “effect” usually quasi
B. physician or exp, however it was not stated that there
C. nurse was no randomization
D. relatives of the patient C. qualitative
*isometric – ex: Kegel’s ; hindi gumagalaw D. descriptive
ang joints 44. In the cause and effect relationship, which is the
38. Andrea needs some physical activity after independent variable?
operation MAINLY to improve healing of which A. Population
parts of the body? Select all that apply. B. Effect
1. Skeletal 2. Integumentary C. Relationship
3. Respiratory 4. Muscular D. Cause
A. 1 only 45. Which of the following is the appropriate
B. 4 only operational definition of wellness? Wellness is
C. 1 and 4 the state of well-being of the participants in
D. 2 and 3 Zumba on their ____________
39. A good example of isotonic exercise for Andrea’s A. social stability
arm are the following, EXCEPT ___________ B. spiritual growth
A. combing the hair C. physical health
B. brushing teeth D. mental balance
C. dressing up
D. closing and opening hand SITUATION 9 -Some teachers in an elementary
40. When doing a range of motion exercise, it is school reported that a high proportion of
important for the nurse to observe Andrea’s children‟s packed lunches contain hotdog or
________. „tocino‟, sweetened drinks and potato chips.
A. Temperature School Nurse Vivian plans to conduct health
B. Color of the skin – circulation education classes among parents on “How to
C. Blood pressure make healthy lunches and why it is important”
D. Respiratory rate
46. For this activity, Nurse Vivian considers using the
behavior change approach to encourage parents
Situation 8 - One of the thrusts of management is to adopt healthy behaviors that will lead to
to encourage more nurses to do research in order improve health. This approach is popular
to contribute to the improvement of the nursing because of the following reasons. Select all that
practice in their work setting. A group of nurses apply.
decided to conduct a study entitled, “Effects of
1. Views health as property of
Zumba as a form of exercise in promoting
individuals
wellness among the health providers in the
2. Sees people as having the
hospital setting.”
capacity to change their lifestyle
41. Which of the following is the independent 3. Assumes that if people do not act
on their health, they suffer the
variable in this study? A. Exercise
consequences
TOPRANK REVIEW ACADEMY INC.
RECALLS 1 EXAM Page 5
4. Does not consider the social and 51. The nursing process facilitates an understanding
environmental factors in which of the scope of challenges inherent in nursing
people live care of clients at risk for ___________ EXCEPT:
A. 1, 2, 3 A. Infection
B. 2, 3, 4 B. Suicide
C. 3 and 4 C. Restraint – an intervention to promote
D. 1 and 2 safety
47. Nurse Vivian finally plans to conduct parent’s D. Injury
classes using the educational approach. This 52. The clients at risk require re-assessment of their
approach is intended to _____. Select all that status on ____________
apply. A. shift basis
1. Provide information B. daily basis
2. Develop the necessary skills C. as necessary basis
3. Provide change to a particular D. injury
direction 53. When a patient is placed in a bed rest, the nurse
4. Help people make an impulsive choice must watch which part of the body most?
A. 1 and 2 A. Liver
B. 1, 2, and 3 B. Head
C. 2, 3, and 4 C. Legs
D. 2 and 3 D. Skin – bed sore
48. To start the health education class and to get 54. The Nursing Practice Standards which Nurse
the parent’s active participation, which of the Amy must let her staff follow includes, EXCEPT
following is the BEST question to ask? ___________.
A. “Did you cook your child‟s lunch today?” A. use a multidisciplinary approval to
B. “Who among you have kids who are enhance client safety as indicated – no
underweight?” need for approval
C. “Can you explain the “Plate Method”?” B. implement emergency measures during fires
D. “What food do you usually prepare and disasters
to your kids lunch?” – asking for personal C. risk elements should not be modified
experience D. use infection control practices that prevent
49. The educational approach to health education or control transmission of the pathogens
states that learning involves three aspects.
Which of the following is NOT included? SITUATION 11 - Applying therapeutic
A. Cognitive – knowledge communication skills is vital in the nurse-client
B. Affective – attitude relationship.
C. Psychomotor – skills
D. Behavioral 55. Which of the following is the BEST SEQUENCE of
50. Since your participants are adult parents, which communication techniques to use during an
of the following are appropriate strategies? assessment interview? Begin with
Select all that apply. ____________.
1. Role playing – for children A. giving a broad opening and move to asking
2. Group sharing focused questions
3. Demonstration B. Giving information and move to asking
4. Return demonstration focused questions – orient pt first
A. 2 and 3 C. 1, 2, 4 C. Asking focused questions and close-ended
questions
B. 2 and 4 D. 1, 2, 3
D. providing information and proceed to stating
observations
SITUATION 10 - Safety in any health agency is a
56. You are caring for Malou, sixteen years old,
must. Nurse Amy wants this to maintain in her
suffering from acute leukemia. You want to
unit.
actively listen to her concerns and understand

TOPRANK REVIEW ACADEMY INC.


RECALLS 1 EXAM Page 6
her meanings. You know that active listening is D. nursing aide
used to _______. 62. In as much as you have not been trained in
A. Treat patient‟s medical problems initiating Intravenous infusion, who among the
B. Recognize the issue that is most following cannot also do it?
important to the client – we listen to A. charge nurse
understand B. staff nurse
C. Help the patient become dependent to the C. nursing aide
nurse in addressing his concerns D. physician
D. Make conclusions regarding client‟s 63. Turning Mr. BC every two hours would be
perception difficult if you do it alone. In order to keep the
57. You ask your newly admitted patient, “What can patient safe, the most number of nurse who can
we do to help you?” You know that this team up with you will be ____________
openended question will _____________. A. Four
A. result in specific information from the patient B. One
B. allow patient to elaborate on his C. Two
response – ventilate anything under the D. Three – chest area, gitna, then feet
sun 64. If Mr. BC would be for dialysis and you need to
C. allow patient to briefly answer question bring him to the dialysis unit, who among the
D. put the patient at ease following, could be the best to help you wheel
58. As you listen to a patient, you need to provide him to the area? A ______________.
feedback that is __________. A. institutional worker – or utility worker
A. behavior-focused and evaluative B. member of the family
B. well-timed and general C. physician
C. general and content-focused D. nurse aide
D. well-timed and content-focused
59. Which of the following statements is TRUE in SITUATION 12- Terry, 15 years old, and a
relation to the use of humor? highschool student, visited the clinic because she
A. Humor should focus on the client’s humanity suspects that she is pregnant. Her pregnancy is
B. Humor is used to build rapport confirmed by the school physician. Terry
C. Constant use of humor can be healing requested nurse Zenaida not to tell her parents
D. Laughter increases neurotransmitters about her being pregnant. She also expressed
desire to terminate her pregnancy.
SITUATION 11 -Mr. BC is confines in the ICU in a
provincial hospital for some complications of his 65. Which of the following ethical concepts apply to
type II diabetes. He is edematous and complained patient Terry‟s case?
of severe chest pain. His vital signs are: A. Bioethics
BP160/98, temperature 37.2C; PR- 90 bpm; RR- B. Deontology – means lang tinitignan ; tama
30 bpm ba yung mean ; walang pakialam sa
consequence basta action tama ; kahit hindi
60. The order of the physician is for immediate maganda consequence basta tama action ok
intubation. For the priority equipment, supplies sakin yan
and material to be used for the procedure, the C. Teleology – tinitignan yung consequences,
nurse should collaborate with the _________. whatever action mali man o tama, basta
A. operating room consequence naging maganda ok sakin yan
B. central supply unit D. Intuitionism
C. anesthesia department 66. Nurse Zenaida should be guided by the steps in
D. emergency department helping Terry make an ethical decision.
61. Morning care had to be done by the ________ Sequence the steps below.
A. nursing student 1. Gather data
B. all the options 2. Make decision
C. staff nurse 3. Identify and clarify the ethical
problem
TOPRANK REVIEW ACADEMY INC.
RECALLS 1 EXAM Page 7
4. Act and assess the outcomes of C. decreased bowel sound – dec peristalsis
decisions D. absent sound
5. Identify options or alternatives 72. The MOST APPROPRIATE nursing diagnosis for
A. 3, 1, 5, 2 & 4 Emma’s case is _________________.
B. 1, 5, 2, 4 & 3 C. A. risk of constipation related to lifestyle
1, 3, 5, 2 & 4 B. perceived constipation related to eating
D. 2, 4, 1, 3 & 5 habits
67. The INITIAL advice of nurse Zenaida to patient C. dysfunctional gastrointestinal motility related
Terry is which of the following? to lifestyle
A. Terry will inform her teacher about her D. constipation related to inactivity
condition and to request the teacher to tell 73. Which of the following are the most simple and
her parents. cost effective interventions that the Nurse can
B. Terry will request the doctor to inform the advise Emma to resolve constipation?
parents 1. Encourage increased fiber in diet
C. Terry will choose a significant other close to 2. Encourage physical activity and
the parents to be the one to relay about her regular exercise
pregnancy 3. Regular time for elimination
D. Advise her that as a minor, her parents 4. Laxative
should be duly informed about her A. 1, 2, & 3
pregnancy – minor pa siya, kailangan alam B. 3 and 4
ng parents niya C. 1, 3 & 4
68. Terry analyzed the advice of Nurse Zenaida and D. 2, 3 & 4
realized the value of life and family. This attitude
74. Health education should include which of the
of realization is termed as _____________
following?
A. Justice
1. Responding to the urge to defecate
B. Ethics and establishing a daily pattern
C. Autonomy 2. Role of dietary fiber and fluid in
D. Values clarification maintaining bowel function
69. Patient Terry’s decision regarding her pregnancy 3. Role of exercise and activity in
is termed as _____________ maintaining bowel function
A. Autonomy 4. Safe and correct use of
B. Justice pharmaceutical agents
C. Veracity A. 2, 3 and 4
D. Fidelity B. 1, 2, 3 and 4
C. 2 and 3
SITUATION 13 - Emma, 65 years of age, has just D. 1, 2 and 3
been widowed a year ago. Her two children are
now all living in the U.S. She used to love cooking SITUATION 14 - Obesity is an emerging concern
but since she is now living alone, she eats meal for patients and nurses. The nurses has an
irregularly. Most of the time, she just watches important role in assessing and evaluating
television. Her chief complaint is constipation. patient‟s physiological status in relation to wight
control
70. When assessing Emma, Nurse Alma should ask
the following, EXCEPT: 75. Nurse Cherry is assessing patients at the
A. Dietary and fluid intake outpatient clinic. Which of the following patients
B. Cultural beliefs – malayo is at risk for health complications related to
C. Lifestyle weight?
D. Bowel pattern A. a thirty three year old who has a body mass
71. When auscultating the bowel sound of Emma, index (BMI) of 24 kg/m
the nurse should be able to hear ___________. B. A fifty six year old who is 6 ft (180 cm) tall
A. loud and gurgling sound and weighs 150 lb (68 kg)
B. increased bowel sound
TOPRANK REVIEW ACADEMY INC.
RECALLS 1 EXAM Page 8
C. A seventy one year old who is 5 ft 4 inches changes in diet and exercise are
(160 cm), weighs 120 lb (55 kg), and carries maintained – best is lifestyle change
most of the weight in the thighs B. the long-term effect of orlistat is not known,
D. a twenty four year old female with a and the drug may cause serious side effects
waist measurement of 30 inches (75 such as heart valve problems
cm) and a hip measurement of 34 C. this drug can cause serious depletion of
inches (85 cm) – waist hip ratio = fatsoluble vitamins and should be used only
waist/hip × 100 ; normal Female: .8 or 80 several weeks
cm D. weight-reduction drugs of any type are used
76. A patient who has been consistently following a for only those who do not have the will
diet and exercise program and successfully power to reduce their intake of food
losing one pound weekly for several months is
weighed at the clinic. However, he has not lost SITUATION 15 - Nurse Beth is working on the
any weight for the last month. Nurse Cherry hospital‟s pediatric unit. She is assigned as a
should first _______________. medication and treatment nurse
A. ask the patient whether there have
been any recent changes in exercise or 80. In preparing to give medications to a
diet patterns – pag may complain, assess preschoolage child, which of the following
muna statements is an APPROPRIATE interaction by
B. discuss the possibility that the patient has Nurse Beth?
reached a temporary weight loss plateau A. “Let me explain about the injection that you
C. Instruct the patient to weigh weekly and will be getting”
record the weights. B. “Do you want to take your medication
D. review the diet and exercise guideline with now?” – initiative vs guilt
the patient C. “Would you like the medication with water or
77. When working with an obese patient who is juice?” – if toddler
enrolled in a behavior modification program, D. “If you don‟t take the medication now, you
which nursing action is APPROPRIATE? will not get better.”
A. Having the patient write down the caloric 81. To determine proper drug dosages for children,
intake of each meal calculations are MOST precisely made on the
B. suggesting that the patient has a reward basis of the child’s __________.
after achieving a weight loss goal A. Weight – also correct but not the ideal or
C. encourage the patient to eat small best answer
amounts throughout the day – focus on B. Height
routine C. body surface area
D. asking the patient about situations that tend D. age
to increase appetite 82. Nurse Beth administered the intramuscular
78. When developing a weight-reduction plan for an medication of Iron by the z-tract method. This
obese patient who is starting a weight loss method is used to ________________. A.
program, which question is MOST important for provide more even absorption of the drug
Nurse Cherry to ask? B. provide faster absorption of the drug
A. “What factors do you think led to your C. prevent the drug from irritating
obesity?” sensitive tissue – displace then tissue
B. “Have you been on any previous diets?” layers are locked > hindi magleleak and drug
C. “What kind of physical activities do you > hindi maiiritate sensitive tissues
enjoy?” D. reduce discomfort from the needle
D. “How long have you been overweight?” 83. The doctor ordered to give a one year old
79. An obese patient asks Nurse Cherry about using patient an intramuscular injection. The
Orlistat (Xenical) for weight reduction. Nurse appropriate and preferred muscle to select for
Cherry advises the patient that _________. this child is the _______________.
A. drugs may be helpful in weight loss, A. Dorsogluteal – you could hit nerves that
but weight gain is likely to recur unless could cause paralysis
TOPRANK REVIEW ACADEMY INC.
RECALLS 1 EXAM Page 9
B. Deltoid B. Pack a cotton pledge (cotton buds) tightly to
C. Ventrogluteal the ear
D. Vastus lateralis – either neonate or infant C. Have the patient remain in the position for
one hour
SITUATION 16 - Patient Cheyenne, three years D. Advice the mother not to let Cheyenne drink
old, was brought to the clinic by her mother due hot water
to ear ache and low-grade fever. While 88. In administering otic medication, Nurse Hazel
performing her physical assessment, Nurse Hazel A. Don the gloves
found Patient Cheyenne‟s right ear to be inflamed B. Perform hand hygiene
and warm to touch. The Pediatrician ordered ear C. Explain procedure and postpone
drops to be instilled to the affected ear. D. Check identification band and proceed

84. Nurse Hazel performs further physical Situation: Nurse Tin is assigned to care for
assessment on Patient Cheyenne BEFORE drug patients with different oxygen delivery systems.
administration, which includes the following,
EXCEPT _______________. 89. As safety precautions for clients receiving
A. Appearance of the pinna and meatus of the oxygen, which among these should Nurse Tin
ear not do?
B. Presence of the interference during the a. Place and arrange substances like acetone,
drug administration alcohol, and oils at the bedside – oxygen is
C. Type of any ear discharge combustible
D. Location and extent of inflammation of the b. Replace woolen blankets with cotton blankets.
ear c. Place a “No Smoking” sign on the client‟s door
85. The MOST APPROPRIATE nursing action before and at the foot or head of the bead.
instilling ear drops to Cheyenne is to ______. d. Ensure that all electric devices are in good
A. Check the medication to be within working order.
room temperature – or warm para 90. Nurse Tin noticed that the patient on nasal cannula
comfortable has encrustations (booger) on her nose. Which
B. Refrigerate the medication for thirty minutes among these is best for Nurse Tin to do first?
C. Fill up the dropper with no more than one a. Call the physician.
millimeter b. Apply a water-soluble lubricant as ordered.
D. Clean the outer surface of the dropper c. Tighten tubing and apply leucoplast to secure
86. In installing any drug into the ears, Nurse Hazel prongs on the nares.
performs in SEQUENCE. Which of the following d. Use a sterile operating sponge to scrape of the
steps? encrustation.
1. Allow the drug to flow into the ears 91. An oropharyngeal airway was inserted to a patient.
slowly Assessment of the mouth may be done as appropriate
2. Tilt the head away from the nurse for the patient‟s condition every:
3. Put a small cotton loosely into the ear a. 1 hour c. 4 hours
4. Wait for 15 minutes before instilling b. 8 hours d. 2 hours
drops on the other ear – if you don’t 92. What is the common position patients are placed into
wait, tatapon gamot sa kabila for suctioning?
5. Instill the ear drops into the affected a. Modified Trendelenburg
ear b. High-Fowler’s with head flexed
A. 1, 4, 2, 5, 3 c. Semi-Fowler’s
B. 2, 5, 1, 3, 4 d. Orthopneic position
C. 4, 1, 5, 3, 2 93. After each suctioning for copious secretions, what
D. 3, 1, 4, 5, 2 should Nurse Tin instruct a conscious patient to do?
87. After instilling medications to the right ear, what a. Hyperventilate c. Assume a tripod position
is the MOST appropriate instruction to the b. Deep-breathe – when you suction, nasusuction
mother? din ang oxygen ng pt
A. Remain in the position for 5 minutes
TOPRANK REVIEW ACADEMY INC.
RECALLS 1 EXAM Page 10
c. d. Avoid unnecessary talking 98. Information gathered by nurses show the importance
of nurse-physician communication. With the current
Situation: Iseminger, Levitt and Kirk (2009) recognition that many medical errors are caused by
defined presencing as the term describing the art communication failure, which of the following is the
of being present, or just being with a client during MOST appropriate intervention?
an “existential moment”. This art of nursing a. Involve a form of communication where 2
presence to promote healing was published in the parties engage in problem solving
journal Nursing Clinics of North America. discussion
b. Organize a conference on medication error
94. Jimmy is a novice nurse assigned in the Cancer participated by nurses, physicians and others
Institute. He maintains his physical presence with the c. Develop a policy where all members of the health
patients, and attends to some of their tasks on the team can use it as a reference
patients‟ behalf. He interacts at a superficial level d. Conduct in-service education for nurses to
with them. According to Osterman and Schwartz- improve competencies to address the issues
Barcott (1996), what level of being present for clients 99. To improve quality client care, the nurses created
is this?
“problem solving committees” headed by senior
a. Presence nursing staff to review standards of care and develop
b. Partial Presence – mababaw lang, not into deep policies and procedures. Its desired outcome is best
with pt shown in:
c. Total Presence
d. Transcendent Presence a. Allowing changes in staff rotation plan to
95. Gigi provides emotional and spiritual presence for the accommodate personal needs of the staff.
patients in the Trauma Ward. Their interactions are at b. More nurses participating in doctor‟s rounds
a transpersonal level, causing a transformative and giving immediate information to doctors
experience. According to Osterman and Schwartz- regarding patient status
Barcott (1996), what level of being present for clients c. Continuous evaluation of nursing practice
is this? and protocols in relation to desired patient
a. Presence outcomes.
b. Partial Presence d. Increasing staff communication like providing a
c. Total Presence bulletin board for sharing information among
d. Transcendent Presence personnel.
96. Which of the following is not a benefit of using nurse
presence in intervening for clients who are helpless,
powerless, and vulnerable? 100. A new wing is being constructed in the hospital
a. Promote healing and the Nursing Director is asked to help design it. To
b. Prevent anxiety – we can’t prevent, only reduce achieve maximum efficiency in carrying out using
or maintain in a certain level activities, the Nursing Director would consider which
c. Create sense of safety of the following conditions to be most helpful?
d. Improved client satisfaction a. Environmental factors such as current economic
97. Which nursing theorist is the most relevant in using status and global issues
this nursing intervention? b. How the structural plan facilities staff interaction
a. Hildegard Peplau c. Jean Watson – and the rituals the nurses use to conduct work
Theory of Human Caring c. Type of equipment and technology and its effects
b. Dorotea Orem d. Madeleine Lehninger on how work tasks are designed and carried out
d. Work flow where equipment, medication,
Situation: Collaboration in the unit is very essential and other items essential for patient care
in relation to patient care. Management functions are stored and positioned
are also carried out in this constant collaboration
by the nurse, in fact, one of the nurses‟ most
important role in patient care is as a collaborator.

TOPRANK REVIEW ACADEMY INC.


RECALLS 1 EXAM Page 11
Situation 2 – The Field Health Services and
Nursing Practice II Information System (FHSIS) is recording and
reporting system in public health care in the
Philippines
Situation 1 – The family of Roxas is fond of dogs.
A vendor who entered the gate without notice is
bitten by one of the pet dogs named Bert. PHN 5. The following are the objectives of the FHSIS,
Cords Attends to the vendor. EXCEPT:
a. Complete the picture of acute and
1. Which of the part of body of the vendor will be chronic disease
the MOST affected in terms of rabies? It is the b. Ensure data recorded are useful and
_________. accurate and disseminated in a timely,
a. Buttocks c. feet easy to use fashion
b. Head d. hand c. Minimize recording and reporting burden
2. To protect the vendor from the dangers of allowing more time for patient care and
rabies, PHN Cords advises him to clean the promotive activities
wound thoroughly with soap and water, consult d. Provides standardized facility-level data
a physician and receive anti-rabies vaccination. base which can be used for more
Which among the following vaccines can provide indepth studies
active immunity? 6. As a nurse, you should know the process of how
1. Purified vero cell these information are processed and
vaccine consolidated. The fundamental block of the
2. Human rabies FHSIS system is the ______________
immunoglobulin a. Family treatment record c. reporting
3. Equine rabies forms
immunoglobulin b. Output record d. target/client list
4. Purified duck embryo 7. The monthly field health service activity report is
vaccine a form used in which of the components of the
a. 1 and 4 c. 3 and 4 FHSIS?
b. 2 only d. 1 only a. Target/client list c. individual health
3. The vendor acquired rabies, what will PHN Cords record
do to protect those who took care of him? He b. Output report d. tally report
should administer____________. 8. In using the tally sheet, the recommended
a. Pre-exposure prophylactic treatment frequency in tallying activities and services
only for the family of Bert is_______.
b. Post-exposure prophylactic a. Weekly c. Monthly
treatment only for the family of the b. Quarterly d. Daily
vendor 9. To monitor clients client registered in long-term
c. Pre-exposure prophylactic treatment to regimen such as the Multi drug Therapy, which
Bert and the vendor‟s families component of the reporting system will be most
d. Post-exposure prophylactic treatment to useful?
Bert and the vendor‟s families a. Output report c. target/client list
4. PHN Cord‟s intervention to protect all residents b. Tally report d. individual health record
who own pets, especially dogs, should be done
by_______. Situation 3 – Nurse Oscar takes care of the Ramos
a. Coordinating with city/ municipal extended family who resides in the house owned
agriculturist for immunization of all pets by Nilda‟s mother, Marta. Nilda tends a variety
b. Coordinating with city/municipal officials store is married to Ramon, a government
to make an ordinance on stray dogs employee. They have four children: Lester, 20
c. Massive campaign to families not to own years old; Gina, 18 years old; Alex, 15 years old;
pets at home and Celine, 12 years old. Lester is a working
d. Massive campaign for responsible student of his second year in computer technology
pet ownership course. Gina is a high school graduate; Alex is in
third year high school and Celine is in Grade six.
There is one year old baby girl who is a daughter
of Gina. Gina, however, could not pinpoint the one

TOPRANK REVIEW ACADEMY INC.


RECALLS 1 EXAM Page 12
who sired her child in as much as she had multiple c. Teach him by step-to-step correct,
sex partners. This angered Ramon. continuous and consistent condom
use
10. Though he recognizes the remorse of his d. Discourage him on having a girlfriend
daughter, Ramon still feels confused regarding and focus more on his studies
the situation. He said he tried his best to support 13. Ramon complains to Nurse Oscar some weird
his family and had always been considerate and behaviour of Nilda. These past few months, she
kind. He and his wife would always give them has decreased sex drive, night‟s sweats and
reminders and advice calmly and never in a mood swings. He also received weird text
nagging manner. But still they failed as parents. message from her such as: “Do you really love
The possible nursing diagnosis of Nurse Oscar of me?”, “What role do I play in your life?” “Do you
his family is_____________. still find me attractive?” The best advice of
a. Interrupted family process Nurse Oscar to Ramon should be________.
b. Impaired parenting a. Give her some money for make-over to
c. Parental role conflict increase her self-esteem and make her
d. Ineffective role performance look attractive to him
11. Nurse Oscar‟s conversation with Gina revealed b. Accompany her to a psychiatric
that the young woman still suffers from a c. Ignore his wife or tell her she is too old
syndrome of failure: failure to complete one‟s to act like a teen-ager
normal growth and development, failure to d. Give reassurance that she is the
complete education, failure to establish a best person who came to his life –
vocation and become independent and failure to pt is possibly experiencing menopausal
have a life. The nurse Oscar‟s intervention to syndrome
this problem is_____________. 14. Apparently, the nurse interventions have
a. Linear approach with regards to the improved family relationships. The members are
individual in the context of the family, now communicating with one another and are
community and culture that will combat excited in preparing for a family affair, which is
shame and guilt the baptism of Beatrice. Nurse Oscar was asked
b. Lay down the foundation of a future by to be the godfather of the child. His best
trusting human association and response is_________.
developing mutual trust initially with the a. Accept and proudly say that Beatrice will
nurse, then the family, and eventually be his 49th godchild
the whole community b. Refuse and make an alibi that he
c. Focus on the factors that will help belongs to another religion
protect Gina towards proximal c. Accept and express gratitude for the
stimuli for healthy growth and trust accorded him by the family
development to develop her d. Politely decline and explain that his
resiliency in confronting current
relationship with the family must
and future problem – pt focused
not go beyond professional
d. Transform interactions among family
members, strengthen specific roles and
Situation 4 – Belinda, the PHN in the Municipality
functions to strengthen family system in
of Tubog, learned from the residents that children
order to eventually cope
and some elderly had been suffering from
12. In one home visit, Nurse Oscar was approached respiratory and skin ailments allegedly due to the
by the 15 year old Alex. He was asking about bad smell curly dark smoke emitting from the
condom use. He said he has a girlfriend with factory nearby. She was invited to the community
whom he is madly in love with but does not assembly that was initiated by the barangay
want her to get pregnant. Nurse Oscar‟s most council.
practical and best advice would be__________.
a. Postpone sex and suggest other ways to 15. The barangay captain asked, “What can you do
expressing love to help solve the problem of Nurse Belinda?”
b. Explain to him the difference between What would be the right response of Nurse
sex and love Belinda?

TOPRANK REVIEW ACADEMY INC.


RECALLS 1 EXAM Page 13
a. “Well, your problem is easy to solve. I phases, the FIRST of which is when the
have here some cough syrup, ointments pregnant woman is________.
for the skin and some antibiotics. I will a. Feels the need to seek professional
distribute this after the meeting.” assistance
b. “Who among you here have children b. Demonstrates self-reliance in caring for
who are suffering from respiratory and herself
skin diseases? How about the adults c. Understands the communication of
who are here? Are you also having the Nurse Lovely regarding the services
same problems?” offered
c. I suppose you gave a lot of thought d. Begins to have feeling belonging
about the problem and its possible 19. Nurse Lovely took note that evaluating the OB
solution. However, treating your staff is an on-going function of management.
children and the elderly is not the Some of the reasons for conducting evaluation
first solution. We have to go to the include, EXCEPT to _____________.
root cause of the problem.” a. Provide an indication of the costs of poor
d. “May I ask you what solution have you quality services
identified for the community problem?” b. Justify the use of resources
c. Dissuade self-evaluation of OB staff
Situation 5 – Nurse Lovely, a newly promoted – evaluate staff reguarly ; dissuade is
senior nurse in Obstetrics ward (OB) is attending opposite of persuade
a seminar on management and leadership in d. Ensure that quality of care is provided
preparation for her work. by the OB staff

16. Nurse Lovely learns the five principles of goal Situation 6 - You are an OB nurse in an out
setting in which the senior nurse must provide patient department of a hospital. You encounter
enough time for OB nurse to improve pregnant women with complication
performance. This is called__________
a. Challenge c. 20. A 35-year old woman, on her 2nd trimester of
feedback pregnancy with insulin-dependent diabetes
b. Commitment d. task mellitus, comes to you for some advice. What is
complexity – pag complex ang task, the PRIORITY message for her at this time?
hindi agad matututunan – learning a. Infants of diabetic mothers are big
curve, give time which can result in more difficult delivery
17. The nurse also learns that continuous training is b. Breastfeeding is highly recommended
a personal as well as an organizational goal. and insulin use is not contraindicated
Choose the statements that are true regarding c. Achievement of optimal glycemic
continuous training control
1. Training employees is an excellent is of utmost importance in
investment and a cost to an preventing congenital anomalies
institution d. Her insulin requirements will likely
2. Continuous training is more of a increase beginning 3rd trimester of
personal responsibility than pregnancy
institutional 21. A 30-year old G6P5 woman at 12 weeks has just
3. Cross training and job rotation begun prenatal care. Her initial laboratory
provide on-going part-time learning reveals that she has human immunodeficiency
experience virus (HIV) infection. What would be a priority
4. Select the best people when hiring evidence-based nurse information for this
employees and invest their retention patient?
through continuous training a. Breastfeeding is still recommended due
a. 3 and 4 c. 1 and to the great benefits to the infant
4 b. Pregnancy is known to accelerate the
b. 1 and 2 d. 2 and 3 course of HIV disease in the mother
18. Noting the importance of Nurse- c. Medication for HIV infection is safe
PatientRelationship, Nurse Lovely reviewed and
Hildegard Peplau‟s Theory which identified three

TOPRANK REVIEW ACADEMY INC.


RECALLS 1 EXAM Page 14
can greatly reduce transmission of HIV d. Provincial Governor
to the infant
d. Breastfeeding will potentiate the Situation 8 – PHN Elfa works in barangay 14 and
transmission of HIV from the mother to 15 in the Municipality of Agoho. One day, a
the child neighbor summons her to attend to a 7-year old
boy with high grade fever.
Situation 7 – Nurse Dora, a nursing staff
applicant, passed both written and oral 26. Upon reaching the house, a local herbolaria,
examinations. Because she knows the head of Nanay Isa was already attending to the boy. She
office, she promised to submit all her credentials said that the boy played near the river and the
after she has “fix things up.” She was appointed bad spirits entered his body. The MOST
as Nurse I with a temporary status until she appropriate remark the nurse make is
submits all her credentials, including her PRC _________________.
license. Her evaluation performance was a. “Go on. Do what you have to do,
satisfactory. After a year though, she had to then I will take over.” – but if there is
renew her PRC registration and identification (ID) possibility of harm, do not allow
card. b. “Nanay Isa, your intervention is entirely
wrong.”
22. What action must the nursing administration do c. “It‟s good you‟re here. You can drive
FIRST? away the spirits that entered the boy‟s
a. Report the matter to the head of body.”
office who had the discretion to d. “You have to be sure that all the evil
appoint the nurse spirits have been driven out of the boy‟s
b. Verify with the Professional Regulation body.”
Commission regarding the status of 27. After a few minutes, Nanay Isa took a big bowl
Nurse Dora of soup and gave to the boy. The BEST remark
c. Confront Nurse Dora and terminate her of the Nurse is _____________.
d. Write a letter to the Civil Service a. “I also drink a soup when I get sick.
Commission for proper action to Nurse How about you, Nanay Isa, do you do
Dora the same?”
23. It was found out that Nurse Dora did not pass b. “The soup could have been better if you
the Nurse Licensure Examinations (NLE). What put lemon grass on it.”
legal action should be filled against her? c. “Come on, tell me why soup must be
a. Dishonesty given to a child with fever
b. Conduct unbecoming of professional d. “That‟s correct. Increasing fluid
c. Malpractice intake will help lower down
d. Misrepresentation – giving false info ; temperature.”
sabi mo nurse ka pero hindi naman 28. Finally, Nanay Isa took out from her pocket a
talaga dried rose flower and place it on the boy‟s
24. In case Dora have medication error during her forehead. How will Nurse Elsa handle this
tour of duty, the head of office can be liable action?
because of the law called ____________. a. Tell her not to use the dried flower
a. Unethical conduct again because it does not have any good
b. Respondeat superior – shared liability effect on the sick
c. Politicking b. Ask for an extra piece of fried rose
d. Res Ipsa Loquitur flower and promise to use it
25. All Nurses must understand that after graduation c. Ask the herboloria the rationale for the
they should pass the NLE. To be registered in intervention
the roster, they should take the Professional d. Leave the intervention as is.
Oath with a ________, EXCEPT. Anyway the intervention is neutral:
a. Member of Sangguniang Panlalawigan not harmful nor beneficial
b. Governor of Philippine Nurse
Association Situation 9 – The local health board established a
c. Member of the Professional Regulatory reproductive health clinic in the main health
Board of Nursing center. Two nurses, Hubert and Irene, were

TOPRANK REVIEW ACADEMY INC.


RECALLS 1 EXAM Page 15
assigned to handle services to address problems d. Purulent discharge which can kill the
related to sexuality, reproductive health and sperm
fertility problems. 33. Nurse Irene further explained that a test used to
determine tubal patency using a radiopaque
29. Nurse Hubert classifies cases according to the material is the __________.
major categories of reproductive tract infections. a. Post-coital infetitlity test
Which of the following is NOT part of such b. Sims Huhner test
classification? c. Friedman‟s test
a. Iatrogenic infections as aftermath of d. hysterosalpingography – dye should
invasive procedures like catheterization reach up to fallopian tube – pag naputol
and intra-uterine device (IUD) insertion agad ang dye (gitna FT or wala pa sa FT
b. Urinary tract infections among male – may bara)
and female patients – urinary and
repro are not similar Situation 10 – Ela, 21 years old, is a law graduate.
c. Sexually-transmitted infections She wants to review for the Bar but thinks she is
d. Endogenous infections resulting from pregnant. She said she has regular menses but
poor personal hygiene does not know when ovulation usually occurs.
30. Irene handles the screening for gonorrhoea This have something to do her fertility period
every two weeks among female sex workers in during her last sexual intercourse with her
the implementation of PD 856. In differential husband.
diagnosis of discharge among infected clients,
which of the following colors discharge will Irene 34. As a nurse, what would you tell Ela regarding
take note to identify gonorrhoea from other ovulation? The ovulation usually corresponds to
causes? the life of the corpus luteum which occurs
a. Greenish yellow as differentiated from approximately _______________.
mucoid white of trichomoniasis a. 14 days after the first day of the
b. Mucoid white as compared to succeeding menstrual
grayishwhite discharge of vaginosis b. 7 days after the first day of the
c. Grayish white as differentiated from succeeding menstrual
mucoid white of chlamydia c. 7 days before the first day of the
d. Yellowish white as compared to succeeding menstrual
Trichomoniasis’ greenish-yellow d. 14 days before the first day of the
31. Nurse Hubert was invited by a women‟s group succeeding menstrual
to give a lecture on healthy sexuality. In the 35. Ela insisted she might have been fertile during
expectation check, he noted that there are the time of sexual intercourse. The nurse
previous misconceptions expressed by the explains that absolute period of fertility is the
participants. Which of the following statements span of time that a woman is likely to be
are correct? pregnant when she engages in unprotected
a. It is the obligation of the wife to give in sex______.
to sex every time he asks for it a. Several days after ovulation (3-4
b. Sexuality is fluid and may change days)
c. Effeminate men are gays b. During ovulation
d. Homosexuality, being gay or lesbian, is c. Immediately after ovulation
an abnormality d. Immediately before ovulation
32. One of the clients was positive to Gonorrhea. 36. The nurse proceeded to take the menstrual
Nurse Irene explained that gonorrhoea and history of Ela to find out if she is likely to be
chlamydia, if left untreated can lead to Pelvic pregnant. Which of the following determines the
Inflammatory Disease (PID). Such condition may date of onset of last menstrual period (LPM)? It
cause infertility due to ______. is the ___________.
a. Foul smelling odor discharge which can a. Duration and character of the LMP
kill the ovum b. Implantation bleeding
b. An unknown cause c. Spotting after the LMP
c. Scarring which can lead to tubal d. Bleeding before the last menstrual
occlusion – sugat, peklat then period (LMP)
magbabara

TOPRANK REVIEW ACADEMY INC.


RECALLS 1 EXAM Page 16
37. The nurse also asked about Ela‟s secondary 41. Nurse Corazon and her group prepared objectives for
amenorrhea that would most likely indicate her plan of action. The following are the characteristics of a
pregnancy. Secondary amenorrhea is cessation good objective. Choose the BEST answer.
of menses for more than_______ months, after a. Futuristic, change-oriented, dynamic
regular menstrual cycle has been established. and systematic
a. Five c. six b. Time-bound, attainable, realistic,
b. Three d. four measureable and specific + client
*primary – 16 y/o never had menstruation centered – The client will ; CSMART
38. The nurse also asked for presence of secondary c. Flexible, accurate, top priority and
dysmenorrhea. Which of the following conditions feasible
is not INCLUDED under secondary d. Time bounded, measurable, change
dysmenorrhea? oriented and continuous process
a. Intra-uterine device (IUD) 42. Nurse Corazon presented the plan in a
b. Pelvic Inflammatory Disease (PID) community assembly for acceptance and approval.
c. Malposition of the cervix Which of the following is the MOST important
d. Absence of any underlying criterion for the plan‟s acceptance and approval
anatomic abnormality manifested by the people?
39. Pediatric patients are prone to falls from hospital a. Tasking the community people
beds which may result to additional cost on the part of b. Volunteerism and willingness of the
the hospital. Which safety measures should be instituted community people
to prevent such incidents? c. Presence of the municipal health officer
1. Restraining them in the assembly
2. Having their mothers or any significant adult d. Huge attendance in the community
with them assembly
3. Advising watchers to put up the bedrails at 43. To combat the delays in attending to childbirths,
all times Nurse Corazon adopts the Integrated Management of
4. Telling them they will be given injection if Childhood Illness in the municipality. This program aims
they will get out of bed alone to __________.
a. 3 and 4 a. Improve and ensure the accessibility and
b. 2 and 3 availability of basic and essential health
c. 1 and 2 care in both public and private facilities
d. 1 and 4 and services
b. Ensure the quality and affordability of
Situation: Municipality “A” made it to a local health goods and services
newspaper headline because of the occurrence of c. Develop, implement, monitor and
some unusual events in just 10 days: 2 pregnant evaluate maternal and newborn
woman died giving birth at home; 2 children died care services (exclude maternal ; IMCI
of dehydration and 2 elderly died of pneumonia. – for under 5 children)
The mayor of the town called for a meeting of the d. Ensuring access to health care services
municipal health office personnel. Public Health 44. In Municipality A, like any other municipalities, one
Nurse Corazon was assigned to make the action MAJOR problem that makes any health service
plan to prevent another similar event to happen. program inaccessible is ___________.
a. Lack of community awareness
40. Which of the following could have been the TOP b. Limited number of health-oriented
contributors to the situation? Select all that apply. programs
1. Postponement of people in seeking medical c. Reliance of government financial
care support
2. Adherence to cultural practices and beliefs d. Lack of ambulance
3. Lack of ambulance and drivers
4. Delay in receiving appropriate and adequate Situation: At 32 weeks gestation, patient
care Angelica, a 16-year-old primigravid, 5 feet tall,
a. 2 and 3 gained a total of 20 pounds, with a one pound
b. 1 and 2 gain in the last 2 weeks. Urinalysis reveals
c. 1 and 4 negative for glucose and a trace of protein. She
d. 1 and 3 came to the clinic for consultation.

TOPRANK REVIEW ACADEMY INC.


RECALLS 1 EXAM Page 17
c. 1, 2
45. Which will BEST describe Angelica‟s risk for d. 1, 2, 3
preeclampsia? 51. The body system of the newborn moves through
a. Proteinuria periods of irregular adjustment in the first six (6) hours
b. Short stature of life. It has two periods namely, the first and the
c. Total weight gain second periods of reactivity. The FIRST period of
d. Adolescent age group – less than 20, reactivity consists of the following characteristics. Select
more than 40 all those that apply.
46. After instructing Angelica how to keep a record of 1. Alertness
daily fetal movement counting (DFMC) at home, the 2. Making sucking sounds
nurse determines that the teaching has been 3. Slow heartbeat
effective when Angelica says she will count the 4. Slow respiratory rate (RR)
number of times the baby moves during which of the a. 1, 2
spans? b. 3, 4
a. 45-minute period after lunch each day c. 1 only
b. 12-hour period each day d. 3 only
c. 30-minute period 3 times a day 52. Which among the following describes Palmar
d. 1-hour period each day – should be Grasp Reflex of the baby? When he ____________.
10 movements a. Is startled as his crib is shaken
47. When teaching Angelica about nutritional needs, the b. Grasps an object and placed on his toes
nurse should emphasize ___________. c. Gets an object placed on his hand
a. High residue diet by closing his fingers
b. Low sodium diet – trigger RAAS > inc BP d. Seems to spit out anything placed in his
c. Regular diet – even if pregnant mouth
d. High protein diet 53. One day, Baby Boy did not want to suck from
48. The nurse mentions some of the conditions the mother’s breast. The doctor ordered intravenous
associated with preeclampsia. Which among the fluids. Which among these complications will Nurse Chari
feedbacks by Angelica to the nurse would warrant anticipate to occur ALMOST IMMEDIATELY?
further explanation? a. Cellulitis
a. Iron deficiency b. Infiltration
b. Physical disability c. Phlebitis
c. Multiple pregnancy d. Burns
d. Diabetes mellitus 54. Later, the pediatrician orders Baby Boy to
49. Angelica was brought to the labor room for undergo exchange blood transfusion because of ABO
magnesium sulfate medication. The PRIMARY blood incompatibility. What should be the BEST
purpose for its administration is to ________. equipment to use in as much as 50mL of blood is
a. Increase the central nervous system‟s ordered?
response to stimuli a. Syringe pump
b. Increase calcium absorption by the b. Soluset with microdip
muscles c. Pulse oximeter – do not use pumps
c. Decrease neuromuscular irritability because blood will hemolyze ; best
d. Reduce peripheral vascular resistance to answer: BT set
lower blood pressure d. Infusion pump
*monitor BURP: BP, Urine Output, RR,
Patella reflex Situation: Nurse Marissa is a clinical instructor
50. Which of the following senses are already developed (CI) of AL University in the North. She is assigned
at birth? Select those that apply. at the OR in a government provincial hospital.
1. Hearing. Marissa is aware of the scope of nursing and her
2. Vision. responsibilities to the school and the hospital.
3. Taste.
4. Smell. 55. What is the FIRST STEP that Marissa should do
a. 1, 2, 3, 4 upon reporting to the OR with her students? She should
b. 1, 3, 4 __________.
a. Orient the students to
the policies of the

TOPRANK REVIEW ACADEMY INC.


RECALLS 1 EXAM Page 18
OR day. From 8:00 o‟clock to 9:00 o‟clock in the
b. Introduce herself and morning, she conducts a health education
coordinate with program to pregnant women. The topic for that
the head nurse day was signs and symptoms that should be
c. Greet the surgeons and reported throughout pregnancy.
anesthesiologists
d. Orient her students to 59. True labor is differentiated from failure labor
the set-up of the Operating Room (OR) because in true labor, contractions will ____________.
56. Marissa understands her responsibilities to the a. Be relieved in the side lying position
school she represents and the hospital she and her b. Bring about progressive cervical
students are affiliated with. Which of the following is dilatation
an INCORRECT statement about student nurses? c. Stop when the client is encouraged to
a. The student nurses are expected to walk
achieve same outcome as license d. Occur immediately after rupture of bag
nurses, that is, to do no harm to clients of water
b. Although student nurses are not yet 60. Based on the characteristics of the Puson family,
licensed they are expected to adhere to Nurse Olive can easily categorize which of the family‟s
the same standards as the licensed developmental stage?
nurse a. Families with pre-school children –
c. The student nurses are the sole majority are PS
responsibility of the CI and school – b. Family with school age children
they are also resp of the hosp c. Beginning family
d. Hospitals or health care agencies may d. Early child-bearing
impose limitations on student nurses‟ 61. In this particular stage, which of the following is
practice and coordination and a most basic concern?
collaboration by the CI and the student a. Emergence of harmony in marital and
nurses must always be practiced. in-law relations.
57. When giving case assignment to student nurses b. Child rearing
at the OR, the CI should inform which of the following c. The couple shifts to adult social
OR personnel? Choose all that apply. interests.
1. OR supervisor d. Balancing time and energy to meet the
2. Chief nurse demands of work and family.
3. Scrub nurse 62. As a young manager, Freda knows that conflict
4. Circulator occurs in any organization. Which of the following
a. 1, 2, 4 statements regarding conflict is NOT true?
b. 3 and 4 a. It may result to poor performance of the
c. 1 and 4 staff.
d. 1 and 2 b. It can be destructive; hence, it should
58. If a student nurse commits a break in asepsis not reach the highest level.
while an operation is on-going, who among the following c. It is not beneficial; hence, it should
persons can call the attention of the student nurse and be prevented at all times.
the entire team? d. It may create a new leader from among
a. The CI only, in as much as she is the the staff.
one directly responsible over the 63. Freda tells one of the staff: “I don‟t have time to
students. discuss the matter with you now”, when the latter asks if
b. The surgeon being the captain of the they can talk about an issue. Which of the conflict
ship. resolution strategies did she use?
c. Any member of the surgical team a. Avoidance
who saw the infraction. b. Smoothing
d. The scrub nurse or the circulating nurse c. Compromise
on-going surgery even if she did not see d. Restriction
the infraction. 64. Which of the following is the BEST action that
she must take?
Situation: Nurse Mitos is assigned at the Maternity a. Quit her job and look for another
Clinic of Hospital M. Every Wednesday is pre-natal employment.

TOPRANK REVIEW ACADEMY INC.


RECALLS 1 EXAM Page 19
b. Disregard what she feels and continue b. Place a small pad under her sacral area
to work independently. c. Put a rolled towel under the Michelle‟s
c. Identify the source of conflict and right hip
understand the source of friction. 70. Nurse Clemens understands that Michelle needs
d. Seek help from the Director of Nursing. more support and encouragement during the PEAK and
65. Freda knows that one of her staff is experiencing MOST painful phase of uterine contractions, which is
burnout. Which of the following is the BEST thing for called ____________.
to do? a. Decrescendo
a. Remind her to show loyalty to the b. Decrement
institutions. c. Increment
b. Ignore her observations; it will be d. Acme
resolved even without intervention. 71. Before the INITIAL vaginal examination
c. Let her staff ventilate her feelings commonly referred to as internal examination, what
and ask how she can be of help. must Nurse Clemens check FIRST?
d. Advise her staff to go on vacation. a. Blood pressure for hypotension or
66. She knows that the performance appraisal hypertension
consists of the following activities, EXCEPT b. Heart rate of Michelle for chest
___________. compression.
a. Setting specific standards and activities c. Abdominal palpitation for signs of fetal
for individual performance. distress.
b. Using agency standards as a guide d. Abdominal palpitation for fetal lie and
c. Focusing activity on the correction position, and fetal heart tone.
of identified behavior – not only on 72. Which of the following is the FIRST step that
weakness ; acknowledge also the Nurse Zyla must consider in preparing a health teaching
strengths plan?
d. Determine areas of strengths and To know _____________.
weaknesses a. Certain factors that may affect the
67. Michelle is a pregnant mother living in Sta. Ana, client‟s learning.
Manila. She has given birth to a twin, one child, and had b. The client‟s personal values and
an abortion. Which of the statements below CORRECTLY expectations
applies, given the obstetrical history of Michelle? c. The client‟s age and educational status
a. Since there has been a twin, Michelle is d. The post partum client‟s needs
G4P4 73. What resources should Nurse Zyla use in
b. Since there was an abortion, Michelle is teaching her postpartum client? Select all that apply.
G4P3 1. Pamphlets
c. Michelle is Gravida 3 Para 2 (G3P2) 2. Audiotapes
d. Michelle is Gravida 4 Para 2 (G4P2) 3. Nurse specialist
G: one twin, one child, one abortion, 4. Videotapes
plus current pregnancy a. 1, 2, 3, 4
P: one twin, one child (abortion is not b. 1 and 17
included) c. 1, 2, 18
68. Michelle is now in her second stage of labor; d. 2 and 22
Cervix 8cm. Nurse Clemens puts her in lithotomy
position. One of the dangers of this position that Nurse Situation: Sandy is a newly hired nurse at the
Clemens should watch for in Michelle is ___________. Neonatal Intensive Care Unit (NICU). She
a. Hyperventilation received an endorsement that Baby Boy A and
b. Hypertension Baby Girl B are ready for discharge. At 5:15 in the
c. Hypotension afternoon she gave the babies to their mothers.
d. Tachypnea Two days after, the mother of “Baby Boy A” came
69. If Michelle has a long second stage of labor and back to complain that she got the wrong baby
needs to be placed in Lithotomy position for a long time, because she got a baby girl instead of a boy.
Nurse Clemens should implement the following safety
measures, EXCEPT ___________. 74. What are the elements that will prove that Nurse
a. Wrap legs with elastic bandages Sandy committed gross negligence? Select all that
a. Put 2 pillows under her head apply.
TOPRANK REVIEW ACADEMY INC.
RECALLS 1 EXAM Page 20
1. That Nurse Sandy has a duty 79. Which of the following health behavior choices are
2. That Nurse Sandy failed to perform that duty ESSENTIAL to promoting health and preventing disease?
3. Due to that duty an injury occurred to her a. Taking prescribed medications at the
patient right time and dosage.
4. Causal relationship as regards the failure the b. Eating the right kind of food,
nurse to perform her duty and the resulting adequate sleep, physical exercise
injury. and effective handling of stress
a. 1, 2, 3 c. Refraining from travelling to countries
b. 3 and 4 with high prevalence of infectious
c. 1 and 2 diseases.
d. 1, 2, 3, 4 d. Smoking cessation and drinking alcohol
75. What could have been done to prevent the regularly.
occurrence of switching of babies? 80. What are the ESSENTIAL factors in the integrative
a. Calling out the name of the mother models of human health?
b. Proper identification of the baby a. Intertwined and interactive, with
using name tags multiple components such as
c. Checking the genitalia physical, psychological and social
d. Identifying the baby by checking their b. Effective only when combined and
footprints integrated with alternative therapies
76. What is the number one element that can prove that c. Generally being attributed to
Sandy is incompetent? Sandy was not able to psychological problems in individuals
perform her job _______________. d. Derived solely from physical
a. Because she is newly hired nurse phenomenon
b. According to the orientation
c. Because she lacks supervision Situation: Mrs. Fabon, 28 years old, has been
d. According to a set standard married for 4 years. She postponed pregnancy to
77. Which of the following should Sandy, being pursue her career as a lawyer. One day, she went
professional nurse, understand as the primary to see an obstetrician because of vaginal spotting
outcome should she be proven guilty of and to find ways of getting her pregnant. After a
incompetence? series of diagnostic examinations, she was found
a. That she can be delisted as member of to have endometriosis.
the APO
b. That she can be terminated from work 81. Which statement BEST describes endometriosis?
by her employer a. It is caused by growth of
c. That she can be charged in the civil endometrial tissues outside the
court uterus.
d. That her license to practice nursing b. It is an infection of the endometrium.
can c. It is caused by abnormal proliferation of
be suspended or revoked for a the endometrial lining.
period of time by the PRC d. It is a major cause of primary
78. Which among the statements are TRUE about health dysmenorrhea.
promotion? It is the process of __________. 82. The nurse performs an assessment on Mrs. Fabon.
1. Enabling individuals and communities to Which of the findings would be indicative of
increase their abilities to control and improve endometriosis? Select all that apply.
their health 1. Spotting after intercourse
2. Enhancing the individuals physical, 2. Persistent dysmenorrhea
psychologic and spiritual well-being 3. Dyspareunia
3. Engaging in correcting the people‟s health 4. Menorrhagia
behavior a. 1, 4
4. Preventing disease from occurring and b. 3, 4
treating it when it occurs a. 1 only c. 1, 2, 3
b. 3 and 4 d. 2, 3, 4
c. 2 only 83. The physician prescribes Danazol (Danocrine) for
d. 1 and 2 Mrs. Fabon. The nurse should tell her that while

TOPRANK REVIEW ACADEMY INC.


RECALLS 1 EXAM Page 21
taking this drug she will experience all these d. Pneumococcus
symptoms, EXCEPT ____________. 89. What is the MOST important advise that Nurse Flor
a. Anovulation should give Client Cedes? It is on risk of
b. Weight gain and edema ____________.
c. Diminished a. Massive bleeding
menstrual flow – eliminate A and D since b. Rupture of the uterus
they are the same c. Ectopic pregnancy
d. Cessation of her menses d. Infection
84. What is the rationale behind
endometriosis interfering with fertility? It is Situation: Mr. and Mrs. Santin have eight children who
because ____________. are all boys. How they would like to be counselled on
a. Of pressure on the pituitary that leads to family planning methods by Nurse Rem.
decreased FHS levels.
b. Of total blockage of endometrial 90. Nurse Rem explains the two forms of hormonal
tissues – pag tumubo sa ibang lugar, contraceptions which are the pill or the Oral
pwede bumara Contraceptive (OC) or the combination of Oral
c. The uterine cervix becomes inflamed Contraceptive (COC). The COC‟s compositions and
and swollen the corresponding actions are, select the TWO
d. The ovaries stop producing adequate correct answers.
estrogen 1. Estrogen acts to suppress the Follicle
Stimulating Hormone (FSH) and Leutinizing
Situation: Cedes is admitted to the hospital Hormone (LH), thus suppressing ovulation.
because of vaginal staining but without pain. Her 2. Progesterone complements the action of
obstetrical history reveals amenorrhea for the last estrogen causing the decrease in the
two months and pregnancy confirmation after her permeability of cervical mucus, thereby
first missed period. limiting sperm motility and access to ova.
3. Estrogen acts to increase FHS and LH, thus
85. Which is MOST likely be the tentative diagnosis? suppressing ovulation.
a. Threatened abortion 4. Progesterone when combined with estrogen
b. Incomplete abortion causes the increase in the permeability of
c. Missed abortion cervical mucus thereby decreasing the
d. Inevitable abortion possibility of tubal transport and
86. A few hours after admission, Client Cedes begins to implantation.
experience bearing-down sensations and expels the a. 3 and 4
products of conception in bed. What should Nurse b. 1 and 4
Flor do FIRST? c. 1 and 2
a. Notify the midwife of the hospital d. 2 and 3
b. Administer a sedative according to 91. Which of the following are the side effects of COC‟s?
physician‟s order Select the CORRECT answers.
c. Check the client’s fundus for 1. Weight gain
firmness 2. Headache
d. Transport her to the Delivery Room 3. Depression
87. Client Cedes asks what is the cause of abortion? The 4. Breast tenderness
MOST appropriate response that Nurse Flor gives is a. 3 and 4
that it is due to _____________. b. 1 and 2
a. Unresolved stress c. 1, 2, 4
b. Physical trauma d. 1, 2, 3, 4
c. Congenital defects *if lactating – progestin only pills: mapupunta sa
d. Embryonic defects baby ang estrogen > hormone replacement therapy
88. Which is the number ONE microorganism that is 92. Nurse Rem advises that the BEST way to lessen the
responsible for infection after a miscarriage? side effects of COC‟s is by __________.
a. Escherichia coli - at rectum the travel a. Changing the routine activities
somewhere else b. Consulting the physician for the side
b. Staphylococcus effects
c. Streptococcus
TOPRANK REVIEW ACADEMY INC.
RECALLS 1 EXAM Page 22
c. Not minding it because it will just 2. Strategy to improve the health of the
subside newborn through interventions before
d. Using different brands of contraceptives conception, at soon after birth and in
postnatal period – di pa nabubuntis
93. Nurse Rem also advises Mrs. Santin to notify the inaalagaan mo na HAHAHAHAHA
health center physician/nurse for the following 3. It aims to reduce global mortality and
symptoms, EXCEPT ______________. morbidity associated with the major causes
a. Severe leg pain of diseases in children and to contribute to
b. Shortness of breath healthy growth and development of children
c. Earache 4. Provides an evidence- based, low cost low
d. Chest pain technology package of interventions that will
save thousands of lives.
94. Mrs. Santin expressed fear in choosing OC or
COCs. She asks about the advantages of using chemical A. 1,2,3
barriers as methods of contraception. Which of the B. 1,3,4
following are the advantages of this type of C. 1,2,4
contraception? D. 1,2,3,4
1. It interferes with sexual enjoyment 97. Interventions carried out that are done within a
2. When used with another contraceptive, they prescribed period would include all except one. Which
increases the other method’s effectiveness should not be included in this category?
3. Various preparations are available in the A. Suctioning the Infant – only when
market APGAR is low
4. They can be purchased without a B. Drying the infant
prescription C. Early skin t skin contact
D. Non- separation from the mother
a. 1 and 2 98. Essential Newborn Care (ENC) gets away with
b. 3 and 4 the usual conscientious use of evidences
c. 1, 2, 3 incorporated into practice. Which intervention is/are
d. 2, 3, 4 removed from practice?
1. Early Bathing
Situation: Mel, Kid, Emma, Gliz, and Trix are close 2. Routine suctioning
friends because they are almost of the same age 3. Bandaging the cord
(8-9 years old), orphans, and out-of-school 4. Application of antiseptic in the cord
youths. Nurse Grace from the Department of stump
Social Welfare and Development (DSWD) is A. 3.4.
tasked to attend to them. B. 2,3,4
C. 1,2,3
95. The nurse‟ responsibility is to check on the D. 1,2,3,4
children’s status/pattern. Which of the following should 99. The Philippines is one of the 42 countries that
be the nurse‟ PRIORITY? account for 90 % of Under Five Mortality worldwide.
More than 82,000 Filipino children under five years old
a. Interaction patterns die every year .Which strategy by the department of
b. Health status – general answer health is sought to attain the objectives of Essential
c. Adaptation status Newborn Care?
d. Developmental patterns A. Integrated management on childhood Illness
B. Unang Yakap Campaign
Situation: Essential Newborn Care is one of the best C. Expanded program on Immunization
strategies to improve newborn care and help reduce D. Basic Emergency Obstetric Care
neonatal morbidity and mortality.

96. Which off the following describes the Essential


Newborn Care protocol?
1. Series of time – bond , chronologically –
ordered, standard procedures that babies
receive at birth

TOPRANK REVIEW ACADEMY INC.


RECALLS 1 EXAM Page 23
NURSING PRACTICE III conduct and values in professional
nursing
Situation 1 – Nurse Louisa is employed in hospital 5. Nurse Louisa has an expired license but
“X” and assigned in the Medical Ward for a year promises to renew her license in due time.
now. The nurse supervisor ordered her to proceed Which of the following violation can she be
immediately to the Surgical Ward as a reliever to charged if she participated in home health care
another nurse who went on emergency sick leave. activity?
She was not oriented in the Surgical Ward and a. Malpractice
the unit was very busy. b. Grave coercion
c. Felony
1. Which of the following is the MOST appropriate d. Negligence
action of Nurse Louisa *conditions wherein you’re not allowed to
a. Request the nurse supervisor to assign a work: suspended, revoked, and expired
more experienced nurse reliever license
b. Refuse the order of the nurse supervisor Situation 2 – Health Education is an area of
and stay put in the medical ward nursing practice when the nurse can be creative
c. Comply with the order of the nurse and independent in the work setting. The
supervisor following questions apply.
d. Request the nurse supervisor to
give her brief orientation before 6. A nurse is developing a Teaching plan for Isabel
compliance 18 year old with Bronchial Asthma. She has an
2. To qualify as an operating room nurse in the order for discharge. Which part of the teaching
Philippines setting, Nurse Louisa should possess plan should be given PRIORITY?
the minimum requirements of _____________. a. Quick relief medicines as ordered
a. Master’s degree holder with valid and b. Avoid contact with fur-bearing pets
current license c. Avoid going to malls
b. Worked in the surgical unit for 8 hours d. Wash bed sheets in warm water
c. RN and has worked abroad 7. Mr Gilbert is for postural drainage. The nurse
d. RN with valid and current license should position the client‟s head at
and Surgical Ward orientation – ______________.
even without surg ward orientation a. No greater than a 25 degree
3. Nurse Louisa is a graduate in the Philippines downward angle
nursing school. As part her professional and b. A 30 degree lateral angle for 25 minutes
personal development, she should attend which c. 25 degree at lateral angle
of the following program? EXCEPT___________ d. A 30 degree downward angle for 25
a. Programs by the Philippine Nurses minutes
Association 8. Nurse Beth is teaching Michel, an asthmatic, on
b. Continuing Professional Development how to use the Spirometer. She should instruct
programs by the Professional Regulation the client to have the
Commission mouthpiece________________.
c. Symposium and forum offered by a. Place into the mouth and have regular
the school breathing
d. Programs of international nurses b. Place into the mouth and have a fast
associations deep breath
4. The PRIORITY objective behind career c. Place into the mouth and inhale
advancement of Nurse Louisa is ____________. slowly
a. Increasing revenue of the service d. Place into the mouth and exhale slowly
providers 9. Nurse Beth is teaching a client on how to use
b. Renew old acquaintances and establish metered dose inhaler to prevent asthmatic
attack while in the hospital. She should instruct
camaraderie
the client to do the following
c. Increased number of networking
__________________ EXCEPT.
activities
a. Keep the head of the bed at 15
d. Updating one’s knowledge, skills,
degree angle – too low, should be
semi fowler’s

TOPRANK REVIEW ACADEMY INC.


RECALLS 1 EXAM Page 24
b. Do oral care after use of the inhaler 14. In the presence of pheochromocytoma, the
c. Use the inhaler before she take her diagnostic test which is expected to be elevated
meals is ___________.
d. Use the inhaler as ordered a. Serum thyroid hormone levels
10. You are conducting health-teaching sessions to b. Albumin globulin test
clients with cardiovascular disorders. Client c. Urine cyclic adenosine mono phosphate
Pedro asks you this question: “Tell me, Nurse, d. 24 hours urine collection for
what I should do with my Hypertension?” The vanillylmandelic acid (VMA) – urine is stored in
best response of a Nurse is____________. the refrigerator
a. “comply with your diet, lifestyle and 15. Palpation, as a modality for physical examination
exercise” is AVOIDED when diagnosed with
b. “strictly follow your prescribed daily pheochromocytoma because this
exercise and smoking cessation” action_______________.
c. “comply with your diet, life style a. Will cause sudden release of
modification and prescribed norepinephrine and severe hypotension
medicines” b. Will cause a sudden release of
d. “include garlic in your meals with cathecolamines and severe hypertension
regulation of alcohol consumption” c. Will displace the location of the tumor
d. Will cause sudden release of
Situation 3 - As a staff nurse in a government epinephrine and severe palpitation
hospital, you have been exposed to varied cases
of clients with endocrine problems. Your nursing Situation 4 – Mr Conrado is being prepared for a
responsibility starts from admission to discharge major surgery. Legal preparation for surgery
which is a domain of your competencies. consists of checking all the required forms for the
operation. Equally important is to make sure that
11. Which of the following questions should you ask the patient is physically, psychologically, and
during an admission interview for a client with a emotionally ready for the procedure.
diagnosis of pheochromocytoma?
a. Do you always feel like you are 16. Informed consent is a process that gives the
suffocating, you want to rest and sleep patient opportunity involved in his or her care.
b. Do you suddenly feel warm and flushed As patient advocate, the nurse ensures the
when you get out of bed following three conditions are present to make
c. Do you notice an increase in your consent valid, EXCEPT:
heart beat? a. Adequate disclosure of the diagnosis by
d. Do you have an increase in urination the physician
lately? b. Comprehension of information by the
12. When the sympathetic nervous system is patient before the operation
stimulated in the case of pheochromocytoma, c. Patient voluntarily giving consent
you expect which of the following signs? d. Forms signed by any close relative
i. Hypertension or watcher
ii. Headache 17. The patient asks you, “What do you think of my
iii. Hyperhidrosis surgeon?” You answered “hmmmmm… he is not
iv. Hypermetabolism really the best one and he seems not to care for
a. 3 and 4 patient...” As a result, the patient switches to
b. 1 only another surgeon. The latter may have grounds
c. 1, 2, 3, and 4 + hyperglycemia to sue you for _____________.
*tumor in adrenal medulla > inc a. Slander - spoken
catecholamines > inc epi, norepi b. Invasion of privacy
13. Which of the following drugs can induce c. Malpractice
hypertensive crisis in Pheochromocytoma? d. Libel - written
a. Tricyclic antidepressant 18. One of your patient‟s visitors whisper to you, “I
b. Corticosteroid hope you will not try to revive my dear friend if
c. Respiratory stimulant her heart stops as she has already suffered a
d. Radio iodine therapy lot.” The correct response is ___________.
a. “That decision is up to the physician”

TOPRANK REVIEW ACADEMY INC.


RECALLS 1 EXAM Page 25
b. “We are all trained in cardiopulmonary an information provided by a
resuscitation” faculty member teaching research –
c. “There is a „ Do not resuscitate‟ order in only those written in formal paper
her chart” 22. Weight is taken as a baseline measurement of
d. “ I understand your concern, but I obese female adolescents as study subjects for a
can’t discuss this matter with you” weight reduction program. This is repeated to
19. According to the Joint Commission, the most note any changes. This pre-test is done
frequently cited factor in sentinel (unanticipated) to___________________.
events that leads to a patient‟s serious physical a. Determine whether the instrument is
or psychological injury is defective
______________________. b. Assess if research design is appropriate
a. Confusion within the health team to the problem identified
b. Miscommunication among health team c. Evaluate whether the instrument is
members defective
c. Incompetence by a team member d. Obtain preliminary data before a
d. Policy changes are not followed by treatment is conducted by the
adequate and consistent staff education researcher – basis for comparison
23. A Nurse researcher is using ACCU-CHEK a
Situation 5 – Conducting Research is one of the monitoring kit to test presence of Diabetes
major roles of the nurses both in hospital and Mellitus among her study subject. How do you
community settings. To be able to develop such classify this type of measurement?
competencies, the nurse has to undergo an actual a. Microbial
conduct of the research process. b. Cytological
c. Physiological
20. Which of the following statements BEST d. Chemical
described a researchable problem? 24. Nurse Joan, wanted to conduct a study using
a. Responses of parents toward having quasi-experimental design. This design will need
children with congenital heart diseases. a __________.
b. The relationship between a. Retrospective evaluation
relaxation technique and relief of b. Field setting for the study
pain of post CABG patients in the c. Comparable group – or control group
surgical coronary care unit. – d. Manipulation of the dependent variable
complete: has relationship, IV, DV, and
population Situation 6 – Kenneth, 16 year old, a foreigner
c. Incidence of medication errors and was admitted in the medical ward due to
reporting practices of Health Care abdominal pain, nausea and vomiting by Nurse
Professional in a teaching hospital Tessie.
d. To what extent do pre-operative
teaching affect the length of 25. In initiating care for patient Kenneth, which of
hospitalization of patients going for the following would be an APPROPRIATE
surgery question to be asked by Nurse Tessie in her
21. Nurse Joan has to undergo literature search for assessment?
her study. She can avail of this from the a. Since this is doctor’s order, you have to drink
following EXCEPT_____________. ice water, instead of hot tea.
a. A summary of research articles that are b. Do you have any books I could read about
relevant to the study people of your culture?
b. A written document published by the c. Do you need to set aside your cultural
investigator herself practices, and comply with hospital rules and
c. Any retrieval from website that will help regulations?
her search for the subject on d. Is there anything I am doing that is not
investigation acceptable to your culture?
d. A description of the scientific study 26. Nurse Tessie respects cultural practices
from integration in her nursing care plan. Which of
the following nursing action is MOST represented
of the culturally competent nurse?

TOPRANK REVIEW ACADEMY INC.


RECALLS 1 EXAM Page 26
a. Help patient Kenneth to learn and a. Give her biscuit to eat – check CBG
understand the language 1st then eat or give D5050 if too low
b. Explain and validate health knowledge b. Do urine testing for sugar
and beliefs of Patient Kenneth with that of c. Provide her warm blanket
the hospital d. Take blood pressure and put her on bed
c. Help Patient Kenneth identify ways to relate rest
more to the culture where they now resides 31. Patient Julia has been classified to have a type II
d. Ask patient Kenneth to help Nurse Diabetes Mellitus. Which of the following is NOT
Tessie in knowing more the culture of a typical manifestation of individuals with this
his origin condition?
27. The family of Patient Kenneth request utilization a. Frequency of urination
of warm compress with banana leaves to Patient b. Increased craving for food
Kenneth. Which of the following is the MOST c. Increased thirst
appropriate response of Nurse Tessie? d. Weight loss
Alternative therapies_________.
a. Cost less than traditional therapies DAY 2: MAY 11, 2021
b. Are use when traditional therapies are
not effective Situation 8 – Mr Dencio, 58 years old is admitted
c. Utilized natural products while to the pay ward because of respiratory problem.
traditional therapies do not The nurse initiated oxygen treatment by mask but
d. Can be effective as traditional the client refuses despite the encouragement by
therapies for some conditions the wife. The client is aware of the benefits of the
28. Patient Kenneth‟s family requests time for treatment.
spiritual healing process in the hospital. This is
allowed by Nurse Tessie and hospital because 32. Which of the following should be given priority?
it______________________. a. Ask the opinion of the wife
a. Gives fulfillment and meaning to the b. Conduct consensus building
patient and family c. Let the attending physician decide on
b. Demonstrate people being responsible for the necessity of the treatment
their life patterns d. Respect the decision of the client -
c. Is non-denominated community service autonomy
d. Formalizes a religious dogma 33. You are taking care of Mr Domingo who is on
29. Positive Practice Environment (PPE) influences the last cycle of radiation therapy for his lung
healing process. Which of the following ways can cancer. You should instruct Mr Domingo to
help Nurse Tessie create a healing environment? a. Brush teeth and gums vigorously after
a. Ensure that relatives and friends visit the meals
patient b. Wait one hour after treatment before
b. Empower clients to make healthy decisions eating
for themselves c. Use mouthwash containing alcohol every
c. Place television in each room of the hospital 2 hours – irritate oral tissues
d. Ensure that staff nurses does not d. Avoid drinking hot fluids – damage
experience burnout oral tissues
34. When the Nurse is assessing a client with
Situation 7 –Julie, 28 years old, has been Congestive Heart failure with pitting edema, the
diagnosed with Diabetes Mellitus. She was Nurse’s documentation will include which of the
advised by her family physician to be admitted to following:
undergo preservation for insulin therapy. Her a. Degree of pitting edema
blood sugar ranges from 200 to 210 mg/dL. At 6 b. Time of indention recovery
am, Nurse Cynthia administered her insulin c. Depth of edema
injection. After 2 hours, the patient complained of d. All of the options
cold clammy perspiration, chilly sensation and 35. Mr. Gabby is with left sided heart failure. Ime‟s
abdominal discomfort. documentation of her assessment findings will
include the following, EXCEPT______________.
30. Which of the following PRIORITY nursing actions a. Dependent edema – RSHF:
should the nurse perform? systemic ; left if lungs
TOPRANK REVIEW ACADEMY INC.
RECALLS 1 EXAM Page 27
b. Pulmonary crackles
c. Difficulty of breathing 39. At what stage of pain mechanism do you classify
d. Cough this pain?
36. A client is on a diuretic therapy. Expected entry a. Perception – nararamdaman niya ;
in patient‟s chart should include the following nociception
information, EXCEPT: b. Modulation
a. Serum electrolytes monitored c. Transmission
b. Intake and output d. Transduction
c. Lasix administered at 8 o‟clock 40. When a client complains of pain less than 6
in the evening – hindi dapat sa months, it is called_____________.
gabi kasi hindi na makakatulog. a. Chronic pain – more than 6 mos, usually
However, can be given if with foley seen in CA
cath because pt doesn’t need to b. Persistent pain
ambulate c. Acute pain
d. Weight is taken before drug is given d. Intermittent pain
41. In order for the nurse to recall the location of
Situation 10 – Myreen, a 42 year old teacher pain, he has to_______________.
with cardiac ailment, nervously informs the a. Asks for onset and duration
doctor that her goiter is getting bigger and b. Mark the painful area in a body
distracts her while swallowing food. The diagram
physician who examined her instructed the c. Asks for facial expression
nurse to admit Myreen and to prepare her for
d. Asks verbal description using pain
surgery after medical clearance.
intensity scale
42. As example of a drug therapy to relieve
37. While interviewing Patient Myreen, she claims
moderate pain is_____________.
that she is anxious for the coming surgery. You
a. Codeine – others are for severe pain
expect the following signs and symptoms when
b. Demerol
one is under stress, EXCEPT___________
c. Methadone
a. Blood loss and weakness
d. Morphine sulphate
b. Increases respiration rate
43. When a client is on prolonged pain therapy, the
c. Decreased mobility
nurse should watch for____________.
d. Pain due to tissue damage
a. Tolerance to drug – yung dating dose
38. Based on your knowledge, Patient Myreen, who
na gumagana sayo, hindi na gumagana
has a history of cardiac illness, should not be
ngayon, kailangaan na taasan
given an enema before surgery. Which of the
b. Allergic reaction to drug
following reasons inhibits the order of enema for
c. Drug resistance
Patient Myreen? Enema____________
d. Addiction to
a. Paralyzes the peristalsis movement
and increases abdominal pain–
enema increases peristalsis by Situation: The nurse at the Operating Room should
be made aware of the functions and implications of
distending the bowel or irritating the
being both a circulating and a scrub nurse. The
intestinal mucosa
following questions apply.
b. Produces vagal stimulation that
is dangerous to cardiac patient
46. The nurse was not able to completely account for the
– vavagal heart rate
sharps that were used during an exploratory
c. Causes constipation and fecal
laparotomy. The surgeon agreed to close the suture
impaction after the surgery – on the even if the situation was mentioned above. It was
contrary, enemas relieve
found out that one needle was still left at the
constipation
peritoneum of the client. The surgeon was accused of
d. Enema results to increased water malpractice because of what happened. Which among
absorption in the bowels the following doctrines explains the scenario above?
a. Res Ipsa Loquitur – the thing speaks for itself,
Situation 11 – Mr MC came to the ER because kita na yung evidence
of sharp troubling pain. After his surgery, he b. Force Majeure
claimed pain is felt even he is asleep.
c. Respondeat Superior

TOPRANK REVIEW ACADEMY INC.


RECALLS 1 EXAM Page 28
d. Subpoena duces tecum d. Adducted and flexed
47. Which among the following is considered as part of the 53. When the patient woke up from anesthesia, she
intraoperative phase? refuses to see her husband. She remarks that she is
a. Patient has decided to have a reconstructive “not the same person, no longer a woman; much,
surgery much less a wife.” Which psychosocial nursing
b. Induction of the anesthesia diagnosis would best describe this situation?
c. Patient having two CTT after a heart transplant a. Altered role performance r/t impaired physical
d. A patient diagnosed to have a constrictive function
pericarditis b. Anxiety r/t surgical removal of the breast
48. Which part of the anesthetic ladder is the patient noted c. Altered body image r/t perception of
to have increased autonomic activity? disfigurement and incapacity
a. Analgesia Phase d. Self-esteem disturbance r/t changing ability to
b. Delirium Phase – dec sensation perform basic wife function
c. Surgical Phase 54. The client was obviously withdrawn although her
d. Danger Stage – overdose recovery from the surgery was uneventful. How can
49. Which among the following is NOT a principle of the nurse be of best help during this period of
surgical asepsis? recovery?
a. Always face the sterile field. a. Allowing the client to have more time to herself
b. Sterile articles unused and unopened are still b. Encouraging the client to have more time to
considered sterile after the procedure. verbalize concerns with her family
c. The sterile field is above the waist level and on top c. Allowing the client to talk with other clients in the
of the sterile field. ward who had the same kind of surgery
d. Eliminate moisture that causes d. Allowing the patient more time to reflect about the
contamination – not moisture but wet effects of surgery
50. Which among the following post-operative 55. During the first 8 hours postoperative, the total
complications is characterized as the collapse of one drainage from the Jackson-Pratt drain attached to the
lung segment or the whole lobe or a number of alveolar wound totaled to 25 mL. What is your next best action?
groups? a. Do nothing as the drainage is expected
a. Pneumoectasis b. Empty the Jackson-Pratt device
b. Atelectasis c. Notify the surgeon stat
c. Pulmonary embolism d. Inform the client that her wound is draining well.
d. Pulmonary Shock 56. A postmenopausal patient is scheduled for a
*if R ang affected – best heard ang sound sa L ; bonedensity scan. You instruct the patient to
tracheal deviation: sa R, kasi itutulak ni L si R ______.
a. report any significant pain to the
SITUATION: You are caring for a married woman physician at least two days before the test.
who underwent modified radical left mastectomy b. ingest 600 mg of calcium gluconate by
mouth for two weeks before the test
51. On admission to Post-anesthesia Care Unit, you read c. consume foods and beverages with a
the OR report which indicates that estimated blood loss high content of calcium for two days before the
during surgery was 1000mL. From the list below, select test.
the MOST objective indicator for the nurse to monitor d. remove all metal objects on the day of
closely. the scan – risk for osteoporsis ; interfere with
a. Changes in vital signs visualization
b. Altered level of consciousness 57. To complete the history and physical
c. Soaked dressing examination of a patient diagnosed with osteoarthritis,
you should assess for presence of _____.
d. Pupillary reaction to light
a. anemia
52. You assisted positioning the client. Her left arm should
be placed in which of the following manner? b. local joint pain
a. Placed above the level of the heart – prev c. weight loss
lymphedema d. osteoporosis
b. Hyperextended away from the chest 58. Ibuprofen (Motrin) 200 mg bid was prescribed
for a patient with left hip pain secondary to
c. Placed at the level of the heart and the hand
osteoarthritis. To minimize gastric mucosal irritation,
below the heart

TOPRANK REVIEW ACADEMY INC.


RECALLS 1 EXAM Page 29
which is a side effect of the drug, what instruction would 57. Nurse Danj should include which of the following
you give the patient as to when to take it? interventions for Lian?
a. on arising a. Increase physical activity to promote intestinal
b. on an empty stomach – for faster absorption activity
c. immediately after a meal – could cause GI b. Instruct the patient to increase intake of raw fruits
bleeding and vegetables
d. at bedtime c. Include high-fiber food choices following the acute
phase of the condition
Situation: Nurse Sylvia is assigned in Out Patient d. Provide Sitz bath for the skin excoriation
Department medical center. Every Wednesday of from bowel movements
the month, the health care services are scheduled 58. What is Nurse Danj’s priority for Lian if the latter
for patients with endocrine disorders. develops fistula from his CD?
a. Fluid and electrolyte balance c. Self-esteem
59. BONUS HAHAHAHA The patient is admitted to needs
the private room for further work-up. You started to b. Pain management d. Skin protection
develop a nursing care plan for Ms. Belen. Which of the 59. Which of the following signs and symptoms may
following nursing diagnosis is APPROPRIATE for her at suggest presence of megacolon (d/t feces) from
this time? antidiarrheal drug use?
a. Disturbed thought processes R/T hyper a. Leukopenia c. Bradypnea
metabolic rate b. Fever d. Hypothermia
b. Constipation R/T gastro intestinal hyper 60. If Lian were a geriatric client, which of these is the
motility first indication of dehydration from fluid volume
c. Imbalanced Nutrition R/T Less body depletion?
requirements a. Tachycardia
d. Activity intolerance R/T decreased metabolic b. Altered mentation – usually mental or altered
rate LOC
60. Once Ms. Belen is confirmed to have c. Hypotension
hypothyroidism, she might experience hematologic d. Fever
compromise.Which of the following is not an expected
hematologic consequence of hypothyroidism? Situation: To carry out management functions in
a. iron and folate deficiency – anemia is d/t any health care setting, it is necessary for the nurse
dec hematocrit to integrate leadership skills that he/she
b. decreased hematocrit developed.
c. oxygen demand is decreased
d. increased Serum cholesterol 61. The organizational chart of a nursing department
illustrates the structure and relationships of the
Situation: Nurse Danj admitted a new patient to nursing leaders and staff of the organization. The
the Medicine Ward: Lianmuel, 30, complains of following are the functions of an organizational chart,
diarrhea for more than two weeks prior to except:
consultation. The diagnosis made was Crohn‟s a. To illustrate centrality of control in the
Disease (CD). A plan of care was made for organization and chain of command
Lianmuel. b. To indicate relationship of leaders to other
management staff
56. Which of these assessments does Nurse Danj expect c. To identify managerial levels
to see in the patient‟s records?
d. To list all functions and duties of the staff –
i. Weight gain of 1kg/day found in job description
ii. Arthralgia 62. Coercing a patient into taking medications by
iii. 10-20 liquid, bloody stools per day threatening punishment could legally be considered
iv. Tenesmus as:
v. Anorexia a. Assault c. Malpractice
vi. Crampy, intermittent pain b. False imprisonment d. Battery
a. i, ii, iii, iv, v, vi c. i, ii iii, iv v 63. The doctor assigned to the patient was also sued
b. iii, iv, v, vi d. ii, iv, v, vi together with the nurses. When it was his turn to take
the stand during the next hearing, he was told to bring
with him a copy of the patient‟s chart. The hearing

TOPRANK REVIEW ACADEMY INC.


RECALLS 1 EXAM Page 30
officer will have to issue what legal order to bring the c. After pain is controlled – Physical exam over
patient‟s chart? nursing health hx
a. Writ of certiorari d. During physical examination
b. Subpoena duces tecum – ducument 70. Rico Y. An was diagnosed to have acute pancreatitis
c. Subpoena ad testificandum – ikaw mag testify from excessive alcohol intake. Which of these is a
d. Writ of quo warranto characteristic of the abdominal pain in acute
64. Mr. Putanesca‟s status has declined after being pancreatitis?
revived yesterday after suffering from arrest. Dr. a. Sharp, intermittent pain
Risotto wrote a DNR order. This order implies that: b. Intense, boring pain
a. The patient need not be given food and water. c. Relieved when in supine position
b. The nurse need not give due care to Mr. d. Worsens when a fetal position is assumed
Putanesca even giving bed bath.
c. The nurses and the attending physician need Situation: Gastrointestinal problems are rampant
not do any heroic or extraordinary measures in the ward Nurse Patricia is assigned in. The
for the patient. following questions apply.
d. The patient need not be given ordinary care so
that his dying process is hastened. 71. Jean is a patient admitted in the ward for her
65. What ethical principle applies when the surgical team hemorrhoid management. The following are
adheres to surgical asepsis during surgical procedure? recommended for caring for Jean, except:
a. Justice c. Maleficence a. Tepid Sitz baths
b. Nonmaleficence d. Beneficence b. Wiping to clean the anal area
c. High-fiber diet
Situation: Nurse Sisley is a nurse in Diliman Doctors d. Use of moistened tissues in cleaning the anal area
Hospital caring for clients with problems in the 72. A patient with colorectal cancer and who underwent
biliary tree. surgery a week ago had colostomy in place. What
assessment finding of the stoma is expected?
66. A patient with complaints of indigestion and RUQ pain a. It is draining bright red blood profusely.
was diagnosed with cholecystitis and underwent an b. It protrudes about 2 centimeters from the
emergency laparoscopic cholecystectomy. Which of abdominal wall
these should Nurse Sisley not include in her post- c. It is dark red and flaccid.
operative care? d. It is reddish pink and dry.
a. Instruct the client about splinting during coughing. 73. Which of these refers to the upper abdominal pain
b. Encourage early ambulation. when eating?
c. Maintain the client flat on bed – ma-aspirate a. GERD c. Achalasia
sa sariling suka kasi may N/V ; should be semi b. Gastritis d. Dyspepsia
fowler’s 74. A patient with duodenal ulcer experiences pain that is
d. Assess the patient‟s O2 saturation level usually aggravated by which of the following?
frequently. i. Ingestion of food
67. Which of these is the best indicator that peristalsis has ii. Caffeine
resumed after the surgery? iii. Fried foods
a. Active bowel sounds c. Drainage from NG suction iv. Spicy food
b. Passage of flatus d. Tympany upon v. NSAID use
percussion vi. Corticosteroid use
*paralytic ileus – absence of GI motility a. i, ii, iii, iv, v, vi c. i, ii, iv, v, vi
68. A T-tube was inserted to maintain patency of the b. ii, iii, iv, v d. ii, iii, iv, v, vi
common bile duct. How should Nurse Sisley position
*relieved by eating – sakit nang mayayaman kasi kain
the client for better drainage?
nang kain HAHAHAHAHAH pag mahirap gastic ulcer kasi
a. Side-lying position c. Semi-Fowler’s
palaging gutom
b. Prone position d. Low Fowler’s
69. Rico Y. An is brought to the emergency department 75. Once peristalsis is established and confirmed after a
because of severe and constant abdominal pain. patient underwent surgery for (diverticulosis – just
Nurse Sisley should conduct comprehensive nursing outpouching, no inflamm) diverticulitis, which among
interview: these does Nurse Patricia expect to be given first?
a. Right upon entrance to the emergency department a. Mashed potato c. Plain gelatin – clear liquid
b. After the vital signs have been taken diet

TOPRANK REVIEW ACADEMY INC.


RECALLS 1 EXAM Page 31
b. Yogurt d. Pudding c. Tell the security guard to keep his screening
device.
Situation: You are caring for patients with various d. Proceed through since the handheld device will
cardiovascular problems. not interfere with the pacemaker.

76. In reading ECG tracings, which of these represents Situation: Hazel, 50 years old, a restaurant
the time required for atrial depolarization, as well as business owner, smokes and drinks in a lot of
the impulse delay in the AV node and the travel time alcohol beverages especially when she is stressed
to the Purkinje fibers? at work. She is obese and has been diagnosed to
a. PR segment c. PR interval be a borderline diabetic. Recently, she is having a
b. QRS complex d. ST segment yellowish discoloration of the skin. She consulted
*conduction system: SA node (pacemaker) > AV her private physician and was advised to be
node > Bundle of His > Purkinje fibers hospitalized because of a suspected fatty liver.
*atrial repo meron pero natatabunan kasi malakas
ang QRS (ventri depo) 86. You are the Nurse-on-duty, when she was admitted.
77. A patient with first-degree AV block will usually You conducted the initial physical examination. The
manifest which of the following in his ECG reading? APPROPRIATE modality in examining an enlarged liver is
a. Prolonged PR interval c. Irregular rhythm to:
b. Widened QRS d. Reverted P waves A. Palpate the liver below the level of the right
Rationale: First-degree AV block occurs when all the rib of the patient
atrial impulses are conducted through the AV node B. Percuss the liver over the right rib cage
into the ventricles at a rate slower than normal. Rate of the patient
is usually dependent on the underlying rhythm, and C. Palpate the liver above the level of the right
rhythm is usually regular. QRS is usually normal, with rib cage of the patient
PR interval > 0.20 seconds. D. Auscultate the liver below the level of the
78. Which of these would characterize a third-degree AV right rib cage of the patient
block? 87. Ms. Hazel was ordered to undergo liver biopsy. The
a. Regular, rapid rhythm nurses responsibility includes the following except:
b. Irregular, rapid rhythm A. Turn the patient on supine position at right edge
c. Regular, normal to slow rhythm of the bed.
d. Irregular, normal to slow rhythm B. Turn the patient to hold her breath while needle is
Rationale: In a complete heart block, the conduction being inserted to the intercostal space
between the atria and ventricles is absent because of C. Turn the patient on the left side after the
electrical block at or below the AV node. Usually, the procedure – apply pressure to prev bleeding
rhythm is regular but the atria and ventricles act D. Secure informed consent
independently. 2 You are aware that Liver biopsy should NOT be
79. Which of these is your priority in caring for a client performed on patient Hazel if she has:
who just had a pacemaker installed? a prothrombin time of 12-15 seconds
a. Encourage patient ambulation. b temperature of 37.2 degree Celsius
b. Minimize patient’s activity – dec cardiac c normal breathing pattern
demand bec body is still adjusting to the d ascites – will not obtain tissue bec of edema
pacemaker 3 For the first 24 hours after liver biopsy the PRIORITY
c. Increase frequency of passive ROM exercises. untoward reactions that you should watch for:
d. Instruct the client to perform pursed-lip breathing a change of mental status
exercises. b stages of hemorrhages – liver is highly
80. Patient Elijah, who has an implanted pacemaker, is vascular
about to enter the airport premise to be able to check c signs of increasing blood pressure
in his flight to Maldives. Knowing the dangers of the d Cyanosis of the extremities
screening devices held by the guards at the airport 4 In planning nursing care for a patient with fatty liver,
entrance, what is the best thing Elijah should do? the nursing intervention should FOCUS on:
a. Request the security personnel to exempt him 1. Limit alcohol intake
from security checking because of his pacemaker. 2. Weight reduction
b. Ask if the security can inspect him manually 3. Lifestyle changes
or through a hand search 4. Reduction of blood sugar
A. 1, 2, 3 and 4 B. 2 only
TOPRANK REVIEW ACADEMY INC.
RECALLS 1 EXAM Page 32
C. 1 and 2 D. 1,2 and 3 and Recording is a part of her job description. The
*if alcohol – LIMIT ; smoking - STOP following questions apply.

Situation: Mr. Ros, a 50 year old was admitted to 96. Nurse Cora made an entry on the patients care
the hospital ward because of abdominal plan: "Goal not Met," patient refuses to undergo breast
enlargement, and is scheduled for exploratory biopsy, patient claimed it is painful. Which of the
abdominal surgery the next day. An informed following should the nurse undertake as an appropriate
consent has to be accomplished. action guided by this recording?
A. Reassess set objectives
91. Who among the members of the surgical team is B. Reassure the patient
responsible to explain the informed consent? C. Notify the physician – respect or accept
A. Surgeon C. Circulating nurse then report para MD ang mag-explain
B. Anesthesiologist D. Medical Director D. Reassess patients behavior
*kung sino ang gagawa 97. The nurse is measuring the patient‟s urine and
92. It is important that Mr. Ros is advised by the is straining it for presence of stone. Which of the
Nurse to have nothing by mouth prior to surgery. If following should the Nurse record in the progress notes
this is a major operation and the schedule of as an objective data? The patient
surgery is at seven in the morning, which of the A. States "I did not see any stone in my urine"
following time for fasting should be observed by the B. Passed out 400 mL of clear urine
patient? C. Claims "I passed few small stones in my
A. 7-8 hours fasting urine"
B. 4-5 hours fasting D. Is complaining of flank pain during urination
C. 9-10 hours fasting 98. Nurse Cora while reviewing the medication
D. 3-4 hours fasting record has observed a handwritten order, which is
*prev N/V, aspiration unclear. What PRIORITY nursing action should the
93. As a surgical nurse, which of the following Nurse do?
interventions are included in your pre-operative A. Call the attention of the doctor who
teaching? prescribed the drug – pa-clarify sa nagsulat
A. Deep breathing exercises – prev B. Refer to the nurse supervisor for the incident
atelectasis report
B. Hand and foot exercises C. Call the pharmacist who dispensed the drug
C. Valsalva Maneuver D. Read the drug reference to check the drug
D. None of the Above 99. The team leader is assessing and recording the
*coughing – prev pneumonia status of their patients for the shift. Their
94. Patient asked the surgical nurse why he had an order documentation should reflect which of the BEST
of nothing per mouth. Your BEST response is to prevent: cardinal principles?
A. Shock C. nausea and vomiting A. Quality and timeliness of nursing care
B. Aspiration D. regurgitation B. Interdisciplinary communication within the
95. Nurse Cleofe would like to assume a patients shift
advocate when the patient for surgery. Which of C. Application of the Nursing
the following is an example of this nursing role? process – either type or form of docu,
A. Nurse ensures patient is may nsg process pa rin
appropriately prepared for surgery – valid D. Compliance with the documentation
and no coercion done standards
B. When the consent is signed by a legal 100. When a patient is with hematemesis, which of the
person following should you read in the charting of nurse Lerma
C. She makes the patient understand the on patient with Gastrointestinal (GI) bleeding?
benefits and risks of procedure A. Small amounts of blood in gastric
D. A nurse makes documentation that the secretions detectable by guaiac test?
surgical procedure is needed B. Bloody vomitus appearing as fresh,
bright red blood, "coffee grounds"
Situation: Documentation is an important aspect of the appearance
nursing process. Nurse Cora is assigned in the medical C. Bleeding coming from esophageal
unit and responsible for a group of patients. Reporting varices observed after surgery

TOPRANK REVIEW ACADEMY INC.


RECALLS 1 EXAM Page 33
D. Black, tarry stools cause by digestion of A. Possible Self-Care Deficit; grooming related
blood to fatigue and muscular weakness
B. Potential for Enhanced Spiritual Well Being
C. Risk for Activity Intolerance related to
prolonged bed rest
NURSING PRACTICE IV D. Constipation related to decreased
activity and fluid intake
Situation 1 – The charge nurse in the medical unit
updates her knowledge on nursing diagnosis. She Situation 2- The nurse provides health education
reviews the terms used to describe clinical to a group of adolescents about pelvic
adjustment. inflammatory disease. (PID).

1. A nursing diagnosis formulated when there is 6. The nurse explains that prevention of PID in
insufficient evidence to support the presence of adolescents is important due to which of the
the problem but the nurse believes the problem following reasons. PID ________:
is highly probable and wants to collect more A. can have devastating effects on the
data is a/an _________: reproductive tract of affected
A. Risk nursing diagnosis – complete data and adolescents
likely to happen B. is easily prevented by compliance to any
B. Possible nursing diagnosis – incomplete form of contraception
data or evidence C. can cause life-threating defects in infants
C. Actual nursing diagnosis born to affected adolescents.
2. Wellness nursing diagnosis Which of the D. Is easily prevented by proper personal
following statements is a WELLNESS nursing hygiene
diagnosis? 7. The nurse explains to the group of adolescents
A. Constipation related to decreased activity that the most common cause of PID is _______:
and fluid intake A. Tuberculosis bacilli
B. Risk for Activity Intolerance related to B. Gonorrhea – tx: Ceftriaxone 250 mg +
prolonged bed rest Azithromycin 1 g
C. Possible Self-Care Deficit; grooming related C. Staphylococcus
to fatigue and muscular weakness D. Streptococcus
D. Potential for Enhanced Spiritual Well 8. The nurse further explains that PID presents the
Being following signs and symptoms, which the
3. Which of the following statements is a adolescent should be aware of:
POSSIBLE nursing diagnosis? A. A hard painless, red and defined lesion on
A. Constipation related to decreased activity the genital area.
and fluid intake B. Small vesicles on the genital area with
B. Potential for Enhanced Spiritual Well Being itching.
C. Possible Self-Care Deficit; grooming C. Lower abdominal pain and urinary tract
related to fatigue and muscular infection.
weakness D. Cervical discharge with redness and edema.
D. Risk for Activity Intolerance related to 9. Which of the following statements is true when
prolonged bed rest teaching adolescents about gonorrhea?
4. Which of the following is a RISK nursing A. Gonorrhea may be contracted through
diagnosis? contact with a contaminated toilet seat.
A. Potential for Enhanced Spiritual Well Being B. The infectious agent for gonorrhea is
B. Possible Self-Care Deficit; grooming related Neisseria gonorrheae.
to fatigue and muscular weakness C. Gonorrhea is most often treated by
C. Risk for Activity Intolerance related to multidose of administration of penicillin.
prolonged bed rest 10. Treatment of sexual partners is an essential part
D. Constipation related to decreased activity of treatment. The nurse further explains to the
and fluid intake adolescents that gonorrhea is highly infectious
5. Which of the following statements is an ACTUAL and it ____:
nursing diagnosis? A. Is limited to the external genitalia

TOPRANK REVIEW ACADEMY INC.


RECALLS 1 EXAM Page 34
B. Can lead to sterility Situation 4 – The ER personnel of hospital X
C. Is easily treated receives a report that a bus traversing a hi-way
D. Occurs rarely among fell off the road. There were several passengers
injured. You are one of the nurses in the ER. You
Situation 3- A 45 year-old female, married with assist in preparing the ER to receive the injured
two children, is admitted in the medical passengers.
department with a diagnosis of osteoarthritis. The
nurse assists in the care of this client. 16. As an ER nurse, one of the most important
assessment skill you need is to prioritize care
11. Which of the following signs and symptoms utilizing the triage system. The triage system is
should the nurse correlate with a diagnosis of used to:
osteoarthritis? A. Rapidly assess clients and classify them
A. Erythema and edema over the affected joints according to age groups,
B. Joints stiffness that decrease with B. Identify and categorize clients so that the
activity + but pain inc least critical are treated last.
C. Anorexia and weight loss C. Identify and categorize clients so that
D. Fever and malaise the most critical are treated first
12. Which of the following factors would MOST likely D. Rapidly assess clients and classify them
increase the joints symptoms of osteoarthritis? according to gender.
A. Emotional stress 17. One of the injured passenger brought in was
B. Obesity – naiipit ang joints unconscious and has multiple trauma. Using the
C. History of smoking 5-level of emergency severity index (ESI), how
D. Alcohol abuse SOON should the injured passenger be seen by a
13. The client asks the nurse what joints are physician?
affected in osteoarthritis. The nurse explains that A. ES4, ES5 (stable), could be delayed – one/no
the joints MOST likely involved in this disease resource needed
are the ________________; B. ES1 (unstable), immediately – requires
A. Cervical spine and shoulder immediate life-saving intervention
B. Fingers and metacarpals C. ES2 (threatened), within 10 minutes – high
C. Hips and knees – weight-bearing joints risk situation or
confused/lethargic/disoriented or in severe
D. Ankles and metatarsals
pain/distress
14. The client asks the nurse what activities she can
D. ES3 (stable), up to one hour – many
perform to effectively decrease the pain and
resources are needed but no danger zone
stiffness in her joints before she starts her daily
vitals
routine. The nurse instructs the clients to
______? (Reference: Agency for Healthcare Research
and Quality)
A. Take a cold bath and rest for a few minutes
18. You do an emergency assessment of one of the
B. Perform range-of-motion exercise and
injured passengers. Which assessment should
apply liniment to the affected joints.
take priority?
C. Take a warm bath and rest for a few
A. Blood pressure
minutes.
D. Stretch all muscle groups. – stretching all B. Breathing pattern
muscle groups is not necessary to decrease C. Airway clearance
the pain and stiffness in the patient‟s joints. D. Circulatory status
15. The physical therapist prescribes a regimen of 19. You take the vital signs of the injured
rest, exercise and physical therapy to the client. passengers. Which of the following vital signs
The nurse understand that this regimen will: indicate thatthe person is experiencing
hypovolemic shock?
A. Provide for the return of joint motion after
prolonged immobility, A. P- 60 beats per minute; RR- 20 breaths per
B. Help prevent the crippling effect of the minute; BP- 100/70 mmHg
disease – limit progression or complications B. P- 80 beats per minute; RR- 22 breaths per
minute; BP- 110/80 mmHg
C. Halt the inflammatory process.
D. Prevent arthritic pain. C. P- 100 beats per minute; RR- 24 breaths per
minute; BP- 140/70

TOPRANK REVIEW ACADEMY INC.


RECALLS 1 EXAM Page 35
D. P- 110 beats per minute; RR- 26 D. Maintain a patent airway.
breaths per minute; BP- 80/60 mmHg – 23. A physical therapist prescribes passive range-
hypo tachy tachy (shock triad) interventions: ofmotion exercises for the legs and assisted
establish IV access, modified ROM exercises for the arms as part of the
trendenlenburg, assess 02, give inotropes – client‟s care regimen. Which observation by the
dopa/dobu nurse would indicate that the treatment is
20. One of the male passengers sustained a effective?
concussion during the accident. He appeared to A. Absence of paralytic drop –
be disoriented and restless. Which of the AKA foot drop
following nursing diagnoses is a priority for the B. Free easy movement of the joints.
care plan? C. Absence of tissue ischemia over bony
A. Sensory or perceptual alteration – health prominences.
threatening problem, medium priority problem D. External rotation of the hips at rest.
B. Dressing or grooming self-care deficit – requires 24. The fracture is repaired by the orthopedic
only minimal nursing support, low priority surgeon. As soon as healing has taken place, daily
problem physical therapy sessions are scheduled that includes
C. Impaired verbal communication - requires only using a tilt table. After the therapist places the client
minimal nursing support, low priority problem at a 45 degree angle, the nurse should monitor the
D. Risk for injury – disoriented or confused d/t client for which of the following? *
concussion (nabagok)
Situation 5 – A 30 year old male fell from a tall A. Hypertension
building while doing construction work. He is B. Pedal edema
conscious and apparently alert but could not move C. Facial flushing
his arms and legs. His co-workers called for the D. Dizziness – d/t orthostatic hypotension
paramedics for emergency assistance. 25. After a month of therapy, the client begins to
experience muscle spasms in his legs. Which of
21. When planning to move a person with possible the following statements is accurate? The leg
spinal cord injury, the primary concern of the spasms _____; *
paramedics would be to __________:
A. Wrap and support the extremities which can A. Indicate that the damaged nerves are
be injured easily. healing.
B. Means eventually, the client will be able to
B. Cushion the back with pillows to ensure
walk again because the damage is not
comfort
permanent.
C. Move the person gently to reduce pain
C. It is a good sign therefore, the client should
D. Immobilize the head and neck to keep on trying to move all the affected
prevent further injury. muscles.
*log roll – turn as one unit D. Occur from muscle reflexes and they
22. The victim is brought to the emergency cannot be initiated or controlled by the
department of hospital X. The physician assesses brain.
the client. The client is confirmed to have spinal
cord injury. Which of the following would be the Situation 6 – Health care personnel including
MOST important nursing intervention during the nurses are part of a team trained to respond to
acute stage of the client‟s care? disasters that may occur in the community. The
A. Monitor vital signs – routine following questions relate to disaster
intervention/assessment preparedness.
B. Maintain proper alignment – this is also
crucial because improper 26. In disaster preparedness, which of the following
alignment/movement can cause further is a responsibility of the nurse? The nurse
damage and loss of neurological function. _____;
Head flexion, rotation or extension should be A. Assists in assigning tasks to members of the
prevented (Saunders, p.2881-2882). emergency response team.
C. Turn and reposition client every 2 hours. – B. Serves as an advocate of the victim of disaster.
during the acute stage, the client must be C. Assumes a leadership role in the command
immobilized to prevent further injury center.
(Saunders, p.1883).

TOPRANK REVIEW ACADEMY INC.


RECALLS 1 EXAM Page 36
D. Assists with triage and emergency health 32. An ideal environment for hospitalized
care needs of the injured or those involved clients should be infection-free. Which of the
in the disaster. following infection protocol is the first line of
27. A disaster preparedness plan is a formal plan of defense against infection?
action for coordinating the response of a health A. Sterilization
care personnel, local government unit and other B. Cleansing of environment
trained personnel in the event of a disaster. C. Disinfecting instruments
Which of the following statement is NOT TRUE D. Hand hygiene
about disaster? 33. A staff nurse delegates routine morning care of a
A. Disasters involve inter agencies working client to a nurse aide. The person MOST
together. responsible for the safe performance of the care
B. Disaster are natural events that causes is the___________.
destruction and devastation. A. Assistant chief nurse for the training and
C. Disasters can be controlled – can education.
mitigate effect only but can’t be prevented B. Nurse aide who perform the care.
from happening C. Staff nurse who delegated the care
D. Disasters result from human-made errors. D. Charge nurse of the unit.
28. Which of the following concepts BEST describes *5 Rights of Delegation:
reverse triage? 1. Right Person
A. Save scarce resource for future use. 2. Right Task
B. Care for victim with life threatening 3. Right Circumstance
problems. 4. Right Direction/Communication
C. Attend to person with disabilities. – 5. Right Supervision
ambulatory 1st
34. The most important reason why a nurse aide
D. Save the ones injured who are in the most
must fully understand how to implement a
critical condition.
delegated procedure is because the nurse aide
29. The nurse in a disaster situation classifies victims
must be able to ______________:
according to the following EXCEPT________? *
A. Urgency of treatment A. Teach another nurse aide the procedure.
B. Perform the procedure quickly.
B. Resources available to treat the victims
C. Severity of injury C. Explain the procedure to the client.
D. Place of treatment D. Complete the procedure safely.
30. A nurse is the first responder in a disaster scene. 35. Nurse Febe makes the daily case assignment of
The nurse should FIRST________: * subordinates and strictly monitors their activities
A. Begin to triage the victims. and decisions. This is an example of what
leadership style?
B. Wait for rescue workers to arrive at the scene.
A. Bureaucratic – the leader relies on
C. Care for victim with life threatening
organizational policies and procedures for
problems – reinforcement ang mag-triage
decision-making (Saunders, p.256).
D. Attend to person with disabilities.
B. Democratic – based on the belief that every
group member should have input into the
Situation 7 – Nurse Febe is a charge nurse in the
development of goals and problem solving; a
oncology unit. She applies principles of leadership
democratic leader acts primarily as a
and supervision in the management of the client’s
facilitator and resource person and is
environment.
concerned for each member of the group
31. Nurse Febe is aware that control of the C. Laissez-faire – a laissez-faire leader assumes
environment is a nurse‟s responsibility. Which a passive, nondirective and inactive
among of the following is the biggest problem of approach and relinquishes part or all of the
hospitalized clients? responsibilities to the members of the group
(Saunders, p.256).
A. Lighting
B. Noise D. Authoritarian/Autocratic
C. Ventilation
D. Hospital bed Situation 8- A 55- year old male is brought to the
emergency ward on a wheelchair by a family
member. He complains of shortness of breath,

TOPRANK REVIEW ACADEMY INC.


RECALLS 1 EXAM Page 37
dyspnea on exertion, palpitation, and cough with B. Psychiatrists
frothy, blood-tinged mucus, fatigue and C. Social workers
weakness. D. Psychologists

36. Based on the information presented, the nurse Situation 9- Ms Benita is a member of ethics
formulates the nursing diagnosis. Which of the committee of a training hospital. Prior to her
following is MOST appropriate? acceptance to the committee, she was tasked to
conduct a research.
A. Activity intolerance related to compromised
oxygen transport system secondary to heart
41. Nurse Benita is guided by basic ethical guidelines
muscle dysfunction.
in the conduct of a research. She should apply
B. Activity intolerance related to compromised
which of the following
oxygen transport system secondary to heart
principles_______________:
failure.
1. Protect the privacy of the subjects.
C. Anxiety related to actual threat to biological
2. Treat the subjects fairly and justly.
integrity secondary to heart failure and
3. Try to avoid injury or harm to the subjects.
death.
D. Ineffective breathing patterns related 4. Get the informed consent of the subjects.
to decreased respiratory depth A. 1 and 3
secondary to pain – remove pain because B. 2 and 3
it is not stated in the situation C. All of the options
37. The nurse assesses the client. Which of the D. 2 and 4
following signs and symptoms would the nurse 42. Nurse Benita understands the principle which
expect of client with heart failure? impose a duty in the researcher to minimize
A. Strong, bounding – should be weak and harm and maximize benefits as the principle of
thready _____________:
B. Bradycardia A. Confidentiality
C. Murmurs B. Justice
38. Normal respiratory rate The nurse administers C. Non-maleficence
oxygen to the client. Which of the following D. Beneficence - “maximize benefits”
statement is TRUE about giving oxygen to
43. The principle of justice connotes which of the
patient with heart failure?
following:
A. Oxygen is administer thru mechanical
A. Fairness and equity – resources based per
ventilation only.
need
B. Oxygen is prescribe to decrease the
B. Confidentiality and anonymity
work of breathing.
C. Fairness and equality – hindi iisipin need ng
C. Oxygen is given to reduce anxiety. bawat isa, resources are distributed equally
D. Oxygen is contraindicated in heart failure.
D. Vulnerability and invulnerability
39. The immediate management for a patient with
44. Which of the following code of conduct has
heart failure are the following, EXCEPT to ____:
endured through the years of ages?
A. Administer intravenous fluids right
A. Code of Ethics for Professionals
away. – limit because of further congestion
B. Sermon on the Mount
B. Monitor heart rate and for dysrhythmia by
using cardiac monitor. C. Ten commandments
C. Provide reassurance of patient. D. Honor Code
D. Assess level of consciousness. 45. Nurse Benita is aware that the participants in a
research should give their consent. The following
40. The nurse is aware that the management of
are vulnerable subjects incapable of giving
heart failure requires the expertise and
informed consent EXCEPT:
consolation with other health disciplines. The
ones who can effectively implement a A. Institutionalized persons
therapeutic regimen to assist clients with the B. Pregnant women
psychosocial aspects that interfere with C. Physically and mentally able person –
maintaining a healthy home life and health care pregnant are allowed however this is the
network are the: umbrella answer
A. Physical therapists D. Children of minor age

TOPRANK REVIEW ACADEMY INC.


RECALLS 1 EXAM Page 38
Situation 10- A 54 year old female was admitted that’s why it is necessary to prepare the
to the emergency department for crushing papers
substernal pain, diaphoresis and nausea. The D. Ensure the client that members of her family
tentative diagnosis of the physician is myocardial can stay with her in the CCU.
infarction.
Situation 11- The nurse in the emergency
46. The nurse in the emergency department department admits a 45 year old female for
understand that the pain experienced by the vomiting blood. According to a family member
client is due to which of the following? A. who accompanied the client, the client had gastric
Myocardial ischemia. ulcer for a several years. The nurse assesses that
B. Irritation of nerve endings in the the client is in shock.
cardiac plexus.
51. Which of the following assessment findings
C. Vasoconstriction because of arterial spasms.
indicate hypovolemic shock?
D. Fear of impending death.
A. Systolic blood pressure is less than 90
*lactic acid accumulation > metabolic acidosis
mmHg.
47. The initial action of the nurse to client is to
_______________: B. Pupils are unequally dilated.
A. Get a complete health history from the C. Respiratory rate is more than 30
client. breaths per minute. – compensate (hypo
tachy tachy) ; in early stage of shock BP is
B. Prepare the client for immediate transfer to
still normal
the coronary care unit.
52. Pulse is less than 60 beats per minute In the
C. Start an intravenous infusion.
early stages of shock, the nurse expects the
D. Administer pain medication prescribed
result of arterial blood gas (ABG) analysis to
by the physician – morphine, analgesics
indicate which of the following conditions
48. The nurse is aware that the area of the heart ____________:
that is most damaged in myocardial infarction
A. Respiratory alkalosis – rapidly removing
________:
CO2
A. Are the heart valves B. Respiratory acidosis
B. Is the right ventricle. C. Metabolic alkalosis
C. Is the left ventricle – biggest ang LV,
D. Metabolic acidosis
biggest need of 02 kaya ito ang pinaka-
53. The physician orders intravenous infusion of
naapektuhan
packed red blood cells and normal saline
D. Is the conduction system. solutions. The nurse assesses the client for
49. The nurse writes a nursing diagnosis for the which of the following _____________:
client. Which of the following is a PRIORITY A. Hypovolemia
nursing diagnosis during the first twenty four B. Anaphylactic reaction – monitor for 15
hours following a myocardial infarction? A. Fluid mins
volume deficit
C. Altered level of consciousness
B. Lack of knowledge about Health
D. Pain
Maintenance
54. The nurse understands that the best indication
C. Impaired Gas Exchange – d/t congestion
that fluid replacement for the client in
> pulmonary edema
hypovolemic shock is adequate is when the
D. High risk for Infection
___________:
50. The nurse admits the clients to the coronary
A. Systolic blood pressure is above 110 mmHg.
care unit. Which of the following actions should
be included in the admission process? B. Diastolic blood pressure is above 90 mmHg.
A. Contact the supervisor of the client in the C. Urine output of 20- 30 mL/Hour. – urine
workplace. output of less than 30 mL/hour is a sign of
hypovolemic shock (Brunner &Suddarth,
B. Keep the client’s family informed of the
p.2043).
client’s progress and status.
D. Urine output is greater than 30
C. Secure information about the client’s
mL/Hour. – best indicator of fluid status is
Philhealth insurance status – B is also
DAILY WEIGHT, however, if performing fluid
correct because our approach is holistic care,
resuscitation it is URINE OUTPUT
however, we are in the admission process

TOPRANK REVIEW ACADEMY INC.


RECALLS 1 EXAM Page 39
55. The physician schedules the client for surgery not working for him. Which of the following
within six hours. The nurse minimize anxiety of actions is MOST appropriate for Nurse Simo?
the client by answering the client‟s questions A. Suggest to the client to try deep breathing to
regarding the surgery in calm manner, keeps the cope with the pain.
client warm, advise the client to be on bed rest B. Explore the nature of the pain and
and dims the lights in the room. The reason for encourage the client to perceive it in a
these interventions is to ________: different way.
A. Increase comfort of the client and her C. Support the client emotionally and tell him
family. – stated na sa situation na may he will receive the next dose of medication
anxiety siya, so psychosocial approach as soon as possible.
B. Minimize oxygen consumption. D. Refer the client to the attending
C. Prevent infection. physician immediately and report that
D. Stabilize fluid and electrolyte balance. the pain medication is not providing
adequate pain relief.
Situation 12- Nurse Simo works in the oncology 60. Nurse Simo assesses a client complaining of
unit. She takes care of cancer patients in pain. She acute pain. The MOST appropriate nursing
is aware that cancer pain management is one of assessment would include which of the
her responsibility. following?
A. The nurses’ impression of clients’ pain.
56. Nurse Simo plans care for a cancer client
B. The clients’ pain rating.
experiencing pain. She is aware that an
important principle of using medication to C. Nonverbal cues from the client.
manage pain is to: D. Pain relief after appropriate nursing
A. Individualize the medication therapy to the interventions.
client. Situation 13- Ms. Cruz is the charge nurse of a
B. Provide the medication as soon as the medical unit. She is responsible for the
client requests for it. management and supervision of the unit.
C. Discontinue the medications periodically to
discourage the development of drug 61. Ms. Cruz observes that one of the female staff
tolerance. nurses is not performing her duties very well.
D. Avoid giving client addictive medications. Which of the following strategies will she
57. Nurse Simo collaborates with the physician in the implement to assist the staff nurse?
development of a drug regimen for the clients. A. Discuss with the staff nurse her
Which of the following medications should be performance and ways she can improve
avoided in the treatment of cancer pain? – verbalize
B. Allow the staff nurse to select own
A. Morphine
assignment.
B. Acetaminophen (Tylenol) – for mild pain
C. Assign the staff nurse several clients with
only
various illness.
C. Meperidine (Demerol) D. Ask the staff nurse to work as an assistant
D. Hydrocodone charge nurse.
58. When titrating (adjusting) a drug for the client in 62. Ms. Cruz notes one of the male staff nurse is
pain, which of the following action is MOST frequently absent and his absence have
appropriate? adversely affected the quality of care given to
A. Ask the physician to include a medication the clients unit. Which of the following would be
order for breakthrough pain. the BEST approach?
B. Follow the physician‟s order for the first 24 A. Talk with the staff nurse regarding the
hours. concern and remind him of the
C. Reassess the client every 8 hours for drug standards of the agency – problem with
effectiveness. individual
D. Seek a new order after 2 doses that do B. Write the staff nurse a memorandum
not achieve a tolerable level of pain regarding his absence.
relief. C. Inform the staff nurse that his absence will
59. One of the clients experience severe, intractable be a ground for termination.
pain and complains that the pain medication is

TOPRANK REVIEW ACADEMY INC.


RECALLS 1 EXAM Page 40
D. Record the absence of the staff nurse in a cancer. The goal of this form of treatment is to
log book. ______:
63. Ms. Cruz assigns a new staff nurse to administer A. Limit the amount of circulating
the medications of a client. Which detail of the androgens. – kasi lumalaki yung prostate
client‟s drug therapy is the staff nurse legally B. Increase prostaglandin level.
responsible to document? The ________. A. C. Increase the amount of circulating
Peak concentration time of the drug. androgens.
B. Safe ranges of the drug. D. Increase testosterone level.
C. Client‟s socio-economic status. 68. The nurse writes a nursing diagnosis of Fear and
D. Client‟s reaction to the drug. Anxiety secondary to the diagnosis of prostate
64. Ms. Cruz decides what is best for a recovering cancer. Which of the following interventions
client and acts on the decision without would be BEST for the nurse?
consulting the client. Ms. Cruz is applying a A. Encourage the client to keep his feelings to
moral principle which is ______________. himself so his family will not be affected.
A. Paternalism – the health care provider B. Establish a nurse patient therapeutic
decides what is best for the client and relationship.
encourages the client to act against his C. Advise the client to have a positive outlook
or her own choices (Saunders, p.206) – relationship.
unconscious and emergency situation D. Provide spiritual support to the client –
B. Beneficence – the duty to do good to others answer always PRAY or SPIRITUAL SUPPORT
and to maintain a balance between benefits if the problem is PSYCHOSOCIAL
and harms (Saunders, p.206)
C. Fidelity – the duty to do what one has Situation 15- The nurse cares for a female client
promised (Saunders, p.206) who is terminally ill and is experiencing pain.
D. Autonomy – respect for an individual‟s right
69. The nurse prepares a care plan for the client.
to self-determination (Saunders, p.206)
The overall goal for the client is ________.
Situation 14- A 60 year old male is admitted to the The client will:
oncology unit. According to the client, he felt a A. Achieve control of pain and discomfort.
growth during a routine digital prostate
B. Receive adequate cerebral oxygenation and
examination. He complains of pain on urination
perfusion.
and frequent urination.
C. Be free from infection.
65. The nurse understands that the function of the D. Receive life sustaining food and liquids.
prostate gland is primarily to ______; 70. The nurse is aware of the document that
A. Regulate the acidity and alkalinity expresses a client‟s wish for life sustaining
environment for proper sperm development. treatment in the event of terminal illness or
permanent unconsciousness. This document is
B. Produce a secretion that aids the
nourishment and passage of sperm. the ______;
C. Secrete a hormone that stimulates the A. No-code
production and maturation of sperm. B. Durable power of attorney – option of client
D. Store undeveloped sperm before ejaculation. or proxy
66. The nurse analyzes the laboratory values and C. Living will – client decides
notes that the serum phosphate level is D. Last will and testament
elevated. This finding indicates which of the 71. The client nears death and requests that no
following: medication be given that would cause a loss of
A. It confirms the diagnosis of prostate cancer. consciousness, including pain medication. The
B. The progression or regression of prostate nurse would promote the best end-of-life care
cancer. for the client by which of the following?
C. The likelihood of metastasis to the A. Discuss the request of the dying client with
bones. family members and respect their wishes.
D. There are complications associated with B. Comfort is the highest priority in this
cancer. situation so give medications as ordered.
67. The nurse knows that hormone therapy is the C. Respect the client‟s wishes and
mode of treatment for a client with prostate withhold pain medications and other

TOPRANK REVIEW ACADEMY INC.


RECALLS 1 EXAM Page 41
medications ordered. – as long as you B. assessment
have explained the possible consequences C. evaluation
D. Be compassionate and give half of dose of D. planning
the medication ordered. 76. Ms.Uman consults with the attending physician
72. Which of the following statement is TRUE about and the anesthesiologist. She advises the primary nurse
terminally ill clients? to proceed with the preparations and to remain alert for
A. Terminally ill clients require minimum any adverse symptoms. Which step of the nursing
physical care. process is this?
B. Health care personnel do not understand A. Assessment
their own feelings about death and dying B. nursing diagnosis
therefore they avoid caring for terminally C. planning
clients. D. evaluation
C. Terminally ill clients have the right to 77. Ms.Uman confers with the client‟s primary nurse
die with dignity. the following morning. Together they determine that the
D. Terminally ill client‟s experiences pain most client is ready for surgery. This step of the nursing
of the time. process is:
73. The dying clients wishes to donate her eyes after A. Evaluation – may napag-usapan na
she dies. Which of the following statements is B. planning
NOT TRUE about organ donation? C. nursing diagnosis
A. Any individual, at least 15 years old of D. assessment
age and of a sound mind may donate a 78. Ms. Uman applies the human relations approach
part of his body to take the effect after in this situation. She is aware that the key to productivity
transplantation needed by the is _________________.
recipient. – minor pa, should be 18 & A. the degree of independence allowed
above B. meeting the objectives of the critical care
B. Sharing of human organs or tissues shall be unit
made only through exchange programs duly C. Firm control of the situation
approved by the Department of Health. D. the behavior of people under direction
C. The choice to donate an organ must be a
written document. Situation 17- Ms. Marcia is a nurse supervisor of
D. Laws do not require the consent of a family three departments in hospital X. She attends an
members to retrieve organs if the donor has orientation seminar on hospital records
expressed his last wish to donate. management.
Section 16- Ms. Uman is a director of the critical
79. Ms. Marcia understands that good client care
care unit of hospital x. She utilizes the nursing
relies on good record keeping. Which of the following is
process to communicate care to the client.
NOT a purpose of hospital record keeping?
74. She is called to the bedside of a client who is A. Records provide evidence of a hospital‟s
scheduled to have laparoscopic cholecystectomy. The accountability.
client‟s pulse is slightly irregular. Ms Uman confers with B. Records are a key source of data for medical
the primary nurse regarding the client’s condition, which research or statistical reports.
step of the nursing process is Ms. Uman applying? C. Records provide data on health information
A. Implementation system.
B. Evaluation D. Records provide personal information
C. Planning – pag nag-uusap ibig sabihin on the physicians and nurses caring for
nagpplano the clients.
D. Assessment 80. Ms. Marcia is aware that when a client is
readmitted to a hospital, the client‟s file is retrieved
75. Ms.Uman calls for a conference with the staff
from the _______________.
members who are attending to the client. They decide to
obtain a 12-lead ECG for a more definitive picture. They A. physician‟s file
conclude that the client has no serious cardiac or B. civil service file
pulmonary problems. Which step of the nursing is in C. master patient index file
effect in this situation? D. hospital library record file
A. nursing diagnosis

TOPRANK REVIEW ACADEMY INC.


RECALLS 1 EXAM Page 42
81. Ms Marcia is aware that when a client is B. Intracellular to extracellular space.
discharged or dies, the following details should be C. Vascular to interstitial space.
entered in the client’s record which is D. Interstitial to vascular space
the_______________. 86. The nurse understands that the fluid shift results
A. Final diagnosis from an increase in the_____________.:
B. Outcomes classification A. Total volume of intravascular plasma
C. Educational attainment B. Total volume of circulating whole blood
D. Religion C. Permeability of capillary walls
82. The following statements are true about patients D. Permeability of the kidney tubules
and hospital records EXCEPT: 87. The client receives fluid resuscitation therapy.
A. Confidential records must be protected against The nurse adjusts the infusion rate by evaluating the
loss, damage , unauthorized access, modification client’s __________:
and disclosure A. Hourly urine output – N: 30-60 mL/hr
B. Patients have the right to confidential treatment B. Daily body weight
of information they provide to health C. Hourly urine specific gravity
professional D. Hourly body temperature
C. Health records are the property of 88. The client receives total parenteral nutrition
community where the patient is treated – (TPN). The nurse understands this therapy will help the
property ng hosp pero ang content property ng client__________.
pt A. Provide adequate nutrition
D. Hospital records maybe released without the B. Ensure adequate caloric and protein intake
patient‟s consent when required in investigation – bawal kumain ang burn pt
for serious criminal offenses
C. Correct water and electrolyte imbalances
D. Allow the gastrointestinal tract to rest
Situation 18 – The nurse assists in the care of a 15
year old female experiencing anaphylaxis due to
Situation 20 – Nurse Merion is a staff nurse in the
insect bite by honeybees.
oncology unit of a tertiary hospital. An activity in
the unit for continuing professional development
83. Upon assessment, the nurse observes the client
is to disseminate information among the
reacting to the insect bites. The following are common
personnel and staff in the unit regarding trends
reactions to an insect sting EXCEPT:
and treatment for cancer. Nurse Marion read an
A. Swelling article entitled “Understanding Colorectal Cancer”
B. Redness which was recently published in a national
C. Appearance of lesions – more of wheal newspaper.
D. Pain
89. According to the Philippine Cancer Facts and
Situation 19 – A 21 year old male is admitted to Estimates for 2010, one of the most common cancer
the burn unit of x hospital. He sustained burns on among men is colorectal cancer. It ranks ______ among
the chest, abdomen, right arm and right leg. all the diseases:
1. EMERGENT PHASE A. First
2. ACUTE PHASE B. Fourth
3. RECOVERY PHASE C. Second
D. Third
84. The nurse assigned to his care anticipates that
the client would be particularly susceptible to which of
Situation: David is a 72 year old who is a smoker
the following fluid and electrolyte imbalances during the
and a social drinker. He consulted the OPD
emergent phase of burn case. because of rectal bleeding. His tentative diagnosis
A. Metabolic acidosis – d/t hyperkalemia is colorectal cancer. He was advised by the doctor
B. Hypernatremia for admission.
C. Hypokalemia
D. Metabolic alkalosis 90. Which of the following would you likely expect as
85. The nurse assesses the client for fluid shifting. a specific complaint of the client during your initial health
During the emergent phase of a burn injury, shifts occur history taking?
due to fluid moving from the_______________. a. Projectile vomiting
A. Extracellular to intracellular space. b. Bouts of hematemesis

TOPRANK REVIEW ACADEMY INC.


RECALLS 1 EXAM Page 43
c. Change in bowel habits – c. 1 and 2
CAUTION US – WARNING SIGNS OF CA d. 1, 2, 3 and 4
d. Passing out white watery stools 97. In order to prevent error in insulin injection
which of the following safety measure should you do?
91. Colonoscopy has been ordered. Which of the a. Leave the drug in the client refrigerator
following is not advisable for the client to do? and ask it when needed.
a. Has to drink the electrolyte laxatives day b. Double check with your head nurse
before the procedure calculations and consider high “alert”
b. Can take PRN medications when taking drugs.
electrolyte solution c. Compute the drug, proceed to the
c. Can have liquid diet before the patient, and administer the drug.
procedure. d. Request the dietary department to keep
d. Chilled electrolyte solution is allowed. the drug until needed.
98. When giving injections to obese client, which of
92. When assessing a client who underwent a the following needle size should you use?
colostomy and the stoma appeared dusky- a. 4 inch needle
purple, it indicates ______. b. 2-3 inch needle – N: 1 inch, if obese use
a. necrosis longer
b. anemia c. 4 ½ inch needle
c. viable stoma d. 1-1.5 inch needle
d. ischemia
94. The nurse should empty the ostomy pouch before it Situation: The ICU nurse assigned to a 60-year old
is 1/3 full. She should _____. acutely ill client with Parkinson‟s disease who was
a. caution the client from straining after surgery. hospitalized frequently. The initial confinement
b. facilitate expulsion of flatus and gas – was due to electrolyte imbalance. The following
pwede sumabog kasi may pressure confinement was due to injury sustained from fall,
he became to have incontinent of stools that
c. prevent dislodging of the skin and ostomy
further lead to development of skin irritation and
drainage
breakdown. Currently he was admitted due to
d. hasten peristalsis and evacuation of feces respiratory infection.
95. Which of the following should the nurse report
IMMEDIATELY to the physician after colostomy? 99. Related literatures included case situations
a. Presence of pink rose to brick red stoma. similar to the case of the client. The nurse is interested
b. No flatus in 24 to 36 hours. in gaining further knowledge that can help the client at
c. Foul smelling odor on the ostomy bag. risk for fecal incontinence. The nurse should use which
d. Skin slightly detached. of the following method to strengthen this report?
a. Historical research method
Situation: The TQM nurse reported to their b. Qualitative research method
director that there are a lot of medication errors c. Experimental research method
committed by nurses and doctors in the hospital d. Quantitative research method
for the past 3 months. They made a decision to 100. The review of literature does not only include
conduct a review of all the cases with these published research studies but also theory. In this case
errors. A clinical enhancement on drug which theory is least related to the study?
administration was strongly recommended by
a. Neuman‟s system model
going back to the basics. The following questions
b. Lazarus‟ theory of stress and coping
apply:
c. Nightingale‟s environmental theory
d. Roy‟s theory of adaptation
96. Which of the following techniques in injection should
the nurse use to minimize pain when administering
imferon (Iron drugs). 1. Z_track technique
2. “Darting” needle quickly
3. Withdraw needle quickly
4. Inject medication quickly
a. 2 and 3
b. 1, 2 and 4

TOPRANK REVIEW ACADEMY INC.


RECALLS 1 EXAM Page 44
Nursing Practice V d. Lumbar puncture (or lumbar/spinal tap)
Fatal: brain herniation
Situation 1 – You are a staff nurse at the 5. Your unit is reviewing national sentinel event
Neurology unit of the Pediatric Ward alerts medication and preparing
recommendations for the unit. Which strategy
1. You have a patient for admission who has been would help reduce medication errors?
diagnosed with Bacterial Meningitis. Which of the i. Utilized only oral syringes to
following type of isolation should you administer oral medication
implement? ii. Eliminate the pediatric satellite
a. Contact precaution pharmacy
b. Universal precaution iii. Increase the steps in the
c. Airborne precaution medical administration procedure
d. Droplet precaution – during 1st 24 iv. Limit the size of I.V. fluids bags
hrs, however, after antibiotic therapy that can be hung on small children
pwede na kahit di naka-isolate v. Reduce the available
2. The attending physician of Mr M who has concentrations or dose strengths of
Cerebral Palsy and a seizure disorder prescribed high alert
Tegretol-XR for him. Master M has a gastrostomy a. 1, 4, 5
feeding tube. The medication prescribed is on b. 2, 3, 4
the hospital‟s “No crush list”. Which of the c. 1, 2, 3
following you should do in order to administer d. 3, 4, 5
the medication? 6. The intervention by a psychiatric nurse that
a. Ask the pharmacist for oral implements the ethical principle of autonomy is
suspension when the nurse______________.
b. Contact the attending physician to a. Stays with the client who is
change the order demonstrating high level of anxiety
c. Dissolve the medication in 30 ml. of b. Intervenes when a self-mutilating client
orange juice attempts to slash wrist
d. Cut the medication into small pieces to c. Explores alternative solutions with
be places to be placed in the feeding the client, who later chooses one
tube alternative
3. You have been observed by your nurse manager d. Suggest that two clients who are
that when you gave the I.V. medication, you fighting be restricted to the unit
disconnected the flush syringe first and then 7. The nurse learns that a patient was admitted
clamped the intermittent infusion device. (should involuntarily on the shift. What assumption can
be clamp > give med > flush) Which of the the nurse make about the patient?
following would be the most effective way to a. For the first 48 hours, he can be given
improve this nursing practice? medication despite his objections
a. Post an evidenced-based article on b. He can leave the ward upon demand
administration of I.V. medication in c. At the time of admission, he was
the neurological unit – best way to
considered to be an imminent danger
convince is through research
to himself or to others
b. Send a group e-mail discussing the
d. He has agreed to accept the treatment
importance of clamping the device first
and participate fully in care planning
c. Create a poster presentation on
8. A male psychiatric nurse receives a call asking
administration of I.V.
whether a certain person has been a patient in
d. Ask each nurse if they are aware that the facility. How should the nurse respond?
their practice is obsolete Nurse_______________.
4. You have a newly admitted patient with bulging a. States that he is unable to give any
fontanels, setting-sun eyes, and lethargy. Which information to the caller
of the following doctor‟s order would you
b. Asks the caller why the information is
question? An order of:
being sought
a. Arterial blood draw
c. Suggests to the caller to speak to Mr X‟s
b. Magnetic resonance imaging doctor
c. Computerized tomography scan

TOPRANK REVIEW ACADEMY INC.


RECALLS 1 EXAM Page 45
d. States that Mr X has been at the facility c. Identify personal feeling that
but gives no further information hinder social interactions –
9. A patient is about to receive electroconvulsive encourage verbalization
therapy (ECT) when the nurse sees that the d. Discuss anxiety-provoking situations
patient has not signed a consent for treatment. 14. The client makes an inappropriate and
Which of the following facts should determine unreasonable report to you. Which of the
the action of the nurse? following principles of good communication skill
a. Verbal consent by the patient is is important for you to use?
sufficient a. Use nonverbal communication to
b. Another patient is needed to witness the address the issue
consent form b. Use logic to address the client‟s
c. Permission is granted by the patient concerns
when he signed the hospital’s admission c. Tell the client that you do not share
form this interpretation – present reality
d. Failure to obtain the patient’s d. Confront the client about the stated
written consent can result to a misperception
lawsuit – any invasive requires consent 15. Which of the following long-term goals is
10. When the nurse is told by the patient that she appropriate for this client?
consented to ECT out of fear of being a. Become appropriately interdependent
abandoned by her husband, what nursing action with others
is required? b. Become involved in activities that
a. Explain that consenting to ECT will make foster social relationships
her husband happy c. Verbalized a realistic view of self
b. Reassure the patient that her decision is d. Take steps to address disorganized
sound thinking
c. Document the patient’s statement
d. Reprimand the husband for coercing his Situation 2 – As a professional nurse you take into
wife consideration the ethico-moral principles in
11. What other traits is expected from a client with providing nursing care.
paranoid personality disorder. The client:
a. Avoids responsibility for health care 16. You released information over the phone to a
actions caller who identified himself as the brother of
b. Is afraid another person will inflict your patient. You found out later that the
harm – passive friendliness – wag brother was out of town. Which of the following
masyado lalapitan, if you need me I’m rights did you violate?
here at the nurse’s station a. Right to privacy
c. Cannot follow limits set on behaviour b. Right to continuity of care
d. Depends on others to make important c. Right to confidentiality
decisions d. Right to respectful care
12. The client discusses current problems with the 17. Your patient is having difficulty making decision
nurse. Which of the following interventions to undergo hip surgery. Which of the following
should have priority in the nursing care plans. nursing actions BEST describes your advocacy
Have the client____________. roles as a nurse?
a. Discuss the use of defense mechanisms a. Protect patient’s autonomy and
b. Look at the source of frustration independence
c. Clarify his thoughts and beliefs b. Communicate patient‟s needs to the
about an event interdisciplinary team
d. Focus on the ways to interact with the c. Advise the client to undergo surgery
others because it is best for her
13. You noticed that the client has impaired social d. Actively support patient‟s decision
skills. Which of the following short-term goals is 18. Mrs Go needs to undergo a hip surgery. She
MOST appropriate for the client? refused to have it done even after the attending
a. Address positive and negative feelings surgeon has explained the procedure thoroughly.
about self Which ethical principle applies in this situation?
b. Obtain feedback from other people a. Autonomy

TOPRANK REVIEW ACADEMY INC.


RECALLS 1 EXAM Page 46
b. Justice following members will be assigned to handle
c. Non-maleficence the activity?
d. Beneficence a. The municipal health officer
19. Informed consent is one of the patient‟s bill of b. The chief of police
rights. One of its requirements is the capacity of c. The municipal mayor
the patient to give it. Which of the following d. The guidance counsellor
elements is/are related to this capacity? (SELECT 25. Which of the following has the MOST influence
ALL THAT APPLY). in the development of alcoholism among the
1. Patient is an youth?
2. He is competent to make a a. Teachers c. parents
choice b. Environment d. peer group
3. He has the freedom to make a
choice Situation 4 – You are a community health nurse
4. He can understand the doing a home visit to a patient with open-angle
consequences glaucoma. The following questions apply to this
a. 1, 2, 3 c. 1, 2, 3, 4 situation.
b. 1, 2, 4 d. 1, 3, 4
20. Another requirement of informed consent is 26. A patient is taking Latanoprost (Xalatan). She
voluntariness. It means, freedom of choice asked you why she was given this medication.
without the following conditions, Your answer is that Latanoprost__________.
EXCEPT__________. a. Moistens the affected eye
a. Force c. consequences b. Prevents the unaffected eye from
b. Fraud d. deceit developing glaucoma
c. Dilates the affected eye
Situation 3 – You are a community health nurse in d. Decreases the intraocular pressure
Municipality B where the incidence of alcoholism in her affected eye – miotic/constrict
among the youth is observed to be increasing. You
want to have an advocacy called “ALCOHOL Situation 5 – Elizabeth has been using contact
WATCH!”. lenses for the last five years. She was admitted to
the eye Unit with a diagnosis of Corneal Ulcer.
21. Initially, who would you invite to organize this
advocacy? (1) Guidance Counsellor of the public 27. Which of the following affects the ability of the
high school; (2) Officer of the local youth office eye to clearly focus? A change in the_______.
(3) Chief of police (4) Municipal health officer; a. Stroma
(5) NGO representative; (6) Parish priest b. Sensory cells of the retina
a. 1, 2, 3, 5 c. 1, 3, 5, 6 c. curvature of the cornea
b. 1, 2, 3, 4 d. 2, 3, 5, 6 d. epithelium
22. Your advocacy will operate on the following 28. Which of the following characteristics make
principle: immune defenses difficult to tend off infections
a. Individual therapy of the cornea? The cornea__________.
b. Group therapy a. Is an avascular tissue
c. Group-help b. Has three major layers
d. Self-help c. Is a vascular tissue
23. Alcohol watch! Will have for its aim: d. Is adjacent to the aqueous humor
a. One at a time step toward total 29. Which of the following is the predisposing factor
withdrawal for Elizabeth‟s condition?
b. A total abstinence from alcohol a. Bacterial infection
c. Total withdrawal from the alcohol b. Prolonged misuse of contact lenses
d. Moderate withdrawal from alcohol c. Malnutrition
*Alcoholic Anonymous (AA) – mismong d. Viral infection
alcoholics 30. Corneal ulcers are considered medical
*Al-Anon – family of alcoholics emergencies. Which of the following nursing
24. You have a plan to have a seminar on the legal actions will be your PRIORITY?
implications of substance abuse. Which of the a. Remove the contact lens

TOPRANK REVIEW ACADEMY INC.


RECALLS 1 EXAM Page 47
b. Prompt referral to the ophthalmologist a. Increased intracranial pressure
for treatment b. Rebleeding
c. Administer eyedrops c. Fluid accumulation in the lungs
d. Pressure dressing applied to both eyes d. Pulmonary emboli
for comfort 36. A non-contrast computed tomography (CT) scan
*surgery: Penetrating keratoplasty (PKP) / was ordered for Mrs H. The purpose of this
corneal transplant diagnostic test is to determine___________.
a. The patency of Mrs H’s airway
Situation 6 – Joseph is an 11-year old Grade IV b. The treatment for Mrs H
pupil in a private school was diagnosed to have an c. The extent of damage to Mrs H’s motor
attention-deficit hyperactivity disorder function
(ADHD) d. If the event is ischemic or
hemorrhagic
31. Which of the following is NOT a MAIN 37. Which of the following is the focus of INITIAL
characteristics of ADHD? assessment for Mrs H.?
a. Passivity c. hyperactivity a. Visual acuity c. motor functions
b. Inattention d. impulsivity b. Emotional status d. airway patency
32. His teacher has observed joseph to have 38. You read in the physician’s notes that Mrs H. has
difficulty waiting for his tutor often batting into contralateral deficits. This means ___________.
conversations and blurts out answers without a. Both sides of the body are involved
waiting for questions to be finished. These are b. A right cerebral vessel is involved
manifestations of_________. c. A left cerebral vessel is involved –
a. Hyperactivity c. passivity opposite side
b. Inattention d. impulsivity d. Deficits will be present below the level of
33. Which of the following is the most important the stroke
nursing consideration in caring for Joseph and
his family? Situation 8 – You are a community health nurse in
a. Establishing a therapeutic Municipality C. You want to improve the primary
relationship with Joseph and his prevention activities in the municipality in relation
family – 1st way to work with pt to mental health.
b. Seeking information regarding mother‟s
course of pregnancy with Joseph 39. In the public health model, which of the
c. Teaching them instructions on giving following is considered the “patient”? The
Joseph‟s medication ___________.
d. Giving them instructions on giving a. Community c. individual
Joseph‟s medication b. Country d. family
40. In assessing mental health needs of the
Situation 7 – Mrs H, a 58-yearold female was community, one of the technique used is
brought in the Emergency Room with the chief epidemiological studies which examine the
complaint of sudden severe headache, numbness incidence and prevalence of mental disorders in
of the right side of her body and difficulty in a defined population. Incidence is the number of
speaking. Oxygen was administered at once to __________.
Mrs H. a. People at risk for mental disorder
b. Existing cases of mental disorder in the
34. The rationale for administering oxygen to total population at a specified point in
suspected stroke patient is __________. the time
a. To prevent further respiratory deficit c. New cases of mental disorder in a
b. To prevent hypoxia (O2 constricts population over a specified period
blood vessels) and hypercapnia of time
(dilates) d. New cases of specific illness in the total
c. To prevent further neurologic deficits population
d. To increase the respiratory function *Prevalence – old and new cases
35. Which of the following risk of complications is 41. Promoting mental health is a primary prevention
lessened through oxygen administration to activity which is consistent with the principle
patient suspected of having a stroke? of________

TOPRANK REVIEW ACADEMY INC.


RECALLS 1 EXAM Page 48
a. Interdisciplinary care b. Ensure patient‟s safety
b. Autonomy c. Return the person to the root of the
c. multidisciplinary care crisis to identify the cause
d. self-help d. Eliminate the stressor
42. The purpose of primary prevention is to lower
the incidence of mental disorder. This can be Situation 10 – Documentation is a registered
done through____________. nurse‟s accountability legal and ethical actions.
a. Reducing the rate at which new cases of
mental disorders develop 47. Charting done by a nurse provides a
b. Early treatment of new cases of mental comprehensive, sequential notes of the following
disorders EXCEPT:
c. Case-finding for population group at risk a. Procedures performed
for mental disorder b. Patient medical diagnosis
d. Program development and c. Information about patient‟s health
intervention for older adults – others status
are secondary d. Relevant health data
48. BONUS HAHAHAHA TAMA LAHAT The following
Situation 9 – You are a staff nurse in a are the importance of proper documentation.
government hospital being transferred to the (SELECT ALL THAT APPLY): 1) facilitates
Psychiatric Unit. You were required to equip interdisciplinary communications; 2) holds vital
yourself by attending the enhancement program information about the patient; 3) reflects quality
on Crisis Intervention. To assess your knowledge and timeliness of nursing care; 4) evidence of
and skills on the subject you were given a pretest. care provided by the doctor and nurses; 5)
utilized as a legal document in a court litigation
43. A crisis that is acute but temporary and due to a. 1, 2, 4 c. 1, 3, 4
external source is__________. b. 2, 3, 4 d. 1, 4, 5
a. Developmental 49. “Compliant with standards” is one of the
b. Transitional characteristics of good documentation. This
c. Traumatic means___________.
d. Dispositional a. A particular order for effective and
*crisis lasts for less than 6 weeks efficient care is followed
44. Which of the following nursing interventions is b. It presents exact and correct details of
the most appropriate for a client who is in the nursing care provided
early state of crisis? c. Specified protocols are dutifully
a. Encourage client to express feeling followed
and emotions related to crisis d. Timely information is provided
b. Require client to be actively involved in 50. Which of the following guidelines in
establishing goals documentation applies when you administer
c. Encourage client to begin the drugs prepared by another nurse?
development of insight a. Document immediately drug given to
d. Ask client to evaluate the situation avoid medication errors
45. Which of the following is NOT an assumption in b. Document that you administered the
the concept of crisis? drug but somebody prepared it
a. Crisis is acute and resolved within a c. There must be existing institutional
short period of time policy that must be observed in
b. All individuals experience a crisis relation to this situation – docu what
c. Crisis is a growth-retarding factor you only prepared, however, depends on
to the emotional development of a hosp policy
person d. Do not document at all
d. Specific identifiable events precipitate a
crisis Situation 11 – Nelson, a 26-year old construction
46. The MAIN objective of crisis intervention is worker, arrives in the Ear Clinic riding a Honda
to_____________ Motorcycle. He is wearing a headset with Mp3
a. Make the person realize his/her player hooked to his belt. Nelson is for hearing
mistakes assessment.

TOPRANK REVIEW ACADEMY INC.


RECALLS 1 EXAM Page 49
51. Pure Tone Audiometry is ordered for Nelson. 57. In administering ear drops, the nurse observes which
Which of the following does this procedure of the following principles?
measure? a. In a child, pull pinna upward and backward.
a. Vestibular portion of the auditory nerve b. Let the ear drops fall on the middle space of the
b. Ear canal volume canal.
c. Structure of the cars c. Lie on the unaffected side to facilitate
d. Hearing acuity absorption.
52. Nelson was found to have Mastoiditis. Which of d. Position unaffected ear uppermost.
the following ear structure is affected? 58. Otosclerosis, a disorder of labyrinth function,
a. Tympanic membrane constitutes which type of hearing loss?
b. Pinna a. Perceptive loss c. Sensorineural loss
c. Eustachian tube b. Conductive loss d. Mixed loss
d. Mastoid air cells *Conductive – reversible, problem sa dinadaanan ng
53. Which of the following is the most common sound (bara/rigid na)
cause of Mastoiditis? *Sensorineural – damage to CN 8 (Vestibulocochlear)
a. Bone tumor irreversible ; ototoxic drugs: Streptomycin,
b. Untreated Otitis Media (inflamm of Furosemide
middle ear) 59. Which of the following is a characteristic sign of acute
c. Meningitis otitis media in children?
d. Mastoid diseases a. Jumping in pain c. Painless
54. Antibiotics have limited use in the actual inflammation
treatment of Mastoiditis because________. b. Ear tugging d. Difficulty awakening
a. Tissue destruction is extensive 60. What makes children more predisposed to chronic
b. It is a long-term treatment otitis media?
c. Antibiotics do not easily penetrate a. Shorter Eustachian tube
the infected bony structure of the b. Horizontal orientation of the ear canal
mastoid c. Primary diaphragmatic breathing
d. Culture has to be done to identify which d. Both A and B
antibiotic is most effective for the
treatment of Mastoiditis Situation: Addiction disorders are unnecessarily
55. Which of the following is the most common common in the modern lifestyle of Filipinos,
treatment for Mastoiditis? especially with the rise of establishments selling
a. Mastoidectomy only products with caffeine. Because of the various
b. Mastoidectomy with tympanoplasty “improvements” in performance, this industry is
still unwavering.
c. Antibiotics with tympanoplasty
d. Antibiotics
61. Which of the following do not have the potential of
addiction, if consumed frequently and in large
Situation: In the PGH Ear Unit, the staff nurse is amounts?
attending to several outpatient clients seeking
a. Chocolate-flavored Cola c. Green tea
follow-up care.
b. Apple juice d.
Common cold preparations
56.The nurse assists in an ear irrigation. Which of the
following statements by the nurse is correct? 62. Caffeine greatly affects which part of the heart, as
reflected in an ECG?
a. “Tilt the head towards the
unaffected ear.” a. Atrium c. Purkinje fibers
b. “Direct the stream of b. Ventricles d.
irrigate at the sides of the ear Interventricular septum
canal.” – wag sa gitna > damage to 63. The nurse suspects caffeine intoxication in a young
eardrum professional if he notes which finding?
c. “After the procedure, lie on the a. Decreased flow of thought and speech
unaffected side to allow the irrigate to b. Psychomotor agitation – agitated dapat pag
soften any hardened mass.” withdrawal
d. “This procedure is allowed for c. Urinary retention
otitis media to clean the canal.” d. Pale face

TOPRANK REVIEW ACADEMY INC.


RECALLS 1 EXAM Page 50
64. In the previous situation of the young professional d. It is a system used to represent the nurses in
intoxicated with caffeine, he suddenly was unable to labor disputes
take any caffeine source for 24 hours already. The 69. A nurse is able to assess that a depressed patient is
nurse expects to note the following findings, except? bleeding from a wrist cut. She immediately direct
a. Headache another staff nurse to call the doctor and send the
b. Difficulty in stimulating nurse aide to gather specific materials to implement
c. Nausea and vomiting measures to control bleeding. Which style of
d. Muscle pain leadership did the nurse use in this situation?
65. The following are the reasons why many people abuse a. Autocratic
caffeine. Choose the exception. b. Democratic
a. Relieve fatigue c. Facilitative
b. Increase mental alertness d. Laissez-faire
c. Both A and B 70. Which strategy should the nurse leader use to
d. Neither A nor B facilitate the performance appraisal conference?
a. Interview with other staff about the employee‟s
Situation: Nurse Fe had been working as a head performance
nurse in the Mental Health pavilion of Davao City. b. Begin the evaluation interview with an
For the past years she had been challenged to cater openended question
to the needs of different mentally ill patients of c. Include personal feelings in the comment
across the continuum of care. sections of the tool for clarity
d. Refrain from adding comments to the evaluation
66. A staff nurse in the psychiatric ward is upset about form
the new female charge nurse who just sits in her office
all day. One of the staff member informs Nurse Fe Situation: The nurse is caring for patients with
about the situation. Which statement by Nurse Fe issues in gender and sexuality. The following
indicates a laissez faire leadership style? situations will apply.
a. “I will schedule a meeting to discuss the concern
of the charge nurse” 71. A nurse was approached by a 23-year-old male who
b. “I hired the new charge nurse and she is doing confessed that he has romantic and sexual affinity
what I told her to do” towards lesbians, gays, transsexuals, bisexuals, a
c. “I will talk to the charge nurse about your straight male and a straight female. The nurse will
concerns and get back to you” help the man identify himself as a? a. Transsexual c.
d. “You and the staff really should take care of Pansexual
this situation on your own” b. Asexual d. Polysexual
67. A volunteer nurse on the ward tells the nurse that one 72. Chester, a 25-year-old male, has been decided that
of the patients on the ward is a neighbor and asks he wants a gender reassignment surgery. He asks the
about the patient‟s condition. Which information nurse as to whom should he ask help regarding his
should the nurse discuss with the volunteer? concern. The nurse will help the client correctly if the
a. Determine how well she knows the patient before nurse refers him to a?
talking with the volunteer a. Endocrinologist – d/t hormones c. Psychiatrist
b. Tell the volunteer the patient‟s condition in b. Psychologist d. Surgeon
layman‟s term 73. Which among the following individuals is
c. Ask the patient if it is all right to talk with the characterized by either a male or a female who
volunteer dresses like a male or a female?
d. Explain that patient information is on the a. Transvestite c. Cross-dresser
need to know basis only b. Transgender d. Transsexual
68. The psychiatric ward is governed by a system of 74. Lea, a 22-year-old lesbian, has decided to have a
shared governance. Which statement best describes gender reassignment surgery and is currently on
an advantage of this system? hormone replacement therapy as she wait for her
a. It guarantees that union will not be able to come surgery. Which among the following features are
into the hospital noticeably the effect of the hormone replacement
b. It makes the manager responsible for sharing therapy?
information with the staff a. Decreasing heart rate
c. It involves staff nurses in the decision b. Increased production of body hairs on areas
making process of the unit with

TOPRANK REVIEW ACADEMY INC.


RECALLS 1 EXAM Page 51
less body hairs than before 79. When attempting to understand the behavior of an
b. Ovarian atrophy older adult diagnosed with Vascular Dementia, the
c. Linea nigra nurse recognizes that the client is probably:
75. After his gender reassignment surgery, Reston felt a. not capable of using any defense mechanisms
that he is not as sexually satisfied as that of when he b. using one method of defense for every situation
was still a male by genitalia. Which among the c. making exaggerated use of old,
following disorders have been noted to be highest familiar mechanism
across males and females who had gender d. attempting to develop new defense mechanism to
reassignment surgery? meet the current situation.
a. Manic Disorders *alam pa ang past nila, ang nawawala ay ang recent
b. Depression memory
c. Schizophrenia 80. The Nurse develops a nursing diagnosis of self care
d. Bipolar Disorder deficit for an older client with Dementia. Which of the
following is the most appropriate goal for this client?
Situation: In the Psychiatric ward nurses are a. The client will be admitted to a long care facility to
discussing the other factors that caused of have activities of daily living needs met
Alzheimer‟s disease (AD). And they all agree that it b. The client will function at the highest level
is a degenerative disease of the brain caused by of independence possible – laging mali ang
gradual death and loss of brain cells resulting to magiging dependent sayo ang pt
progressive and irreversible Dementia. c. The client will complete all activities of daily living
independently within one (1 ) hour time frame
76. Which of the following nursing intervention is most d. The Nursing staff will attend to all the client‟s
helpful in meeting the needs of an older adult activities of daily living needs during the
hospitalized with the diagnosis of Dementia of the hospitalization
Alzheimer‟s type?
a. providing a nutritious diet high in carbohydrates Situation: The diverse Neurologic disorders
and protein present a unique challenges of nursing care. The
b. simplifying the environment as much as Nurse must have a clear understanding of the
possible while eliminating the need for pathologic processes for appropriate nursing
choices management. Nurse Marco is attending to clients
c. developing a consistent nursing plan with fixed in the ward with Multiple Sclerosis.
time schedules to provide for emotional needs
d. providing an opportunity for many alternative 81. Which statement by a client with Multiple Sclerosis
choices in the daily schedule to stimulate interest (demyelination of myelin sheaths) indicates to Nurse
77. The nurse recognizes that Dementia of the Marco that the client needs further teaching?
Alzheimer‟s type is characterized by: a. “I use a straw to drink liquids.”
a. aggressive acting-out behavior b. “I will take a hot bath to help relax my
b. periodic remissions and exacerbations muscles.” – pwedeng nasusunog ka na, hindi mo
c. hypoxia of selected areas of brain tissue pa alam
d. areas of brain destruction called senile c. “I plan to use an incontinence pad when I go out.”
plaques d. “I may be having a rough time now, but I hope
78. A 75-year-old man with the diagnosis of Dementia has tomorrow will be better.”
been cared for by his wife for 5 years. For the past 2 82. A recently hospitalized client with Multiple Sclerosis is
years he has not spoken and incontinent of urine and concerned about generalized weakness and a
feces. During the last month he has changed from fluctuating physical status. What is the priority nursing
being placid and easygoing to agitated and intervention for this client? a. encourage bed rest
aggressive. He is admitted to a Psychiatric hospital for b. space activities throughout the day
treatment with Psychopharmacology. Which is the c. teach the limitations imposed by the disease
priority nursing care while this client is in the d. have one of the client‟s relatives stay at the
psychiatric facility? bedside
a. managing his behavior 83. Marco is excited to be assigned in a Neuro –Ward after
b. preventing further deterioration his extensive training. He is preparing to conduct a
c. focusing on the needs of the wife Neurologic examination. What nursing intervention is
d. establishing on the needs of the wife anticipated for a client in the plateau phase of Guillain-
Barre syndrome?

TOPRANK REVIEW ACADEMY INC.


RECALLS 1 EXAM Page 52
a. providing a straw to stimulate the facial muscles 89. Which of the following is the first line treatment
b. inserting an indwelling catheter to monitor for acute mania?
urinary output A. Imipramine
c. encouraging aerobic exercises to avoid muscle B. Sodium valproate
atrophy C. Electro-convulsive therapy
d. administering antibiotic medication to prevent D. Lithium carbonate
pneumonia *normal range: 0.5-1.5
84. Mr. Rod a 48 year old client carpenter admitted after *therapeutic range: 0.6-1.2
a spinal cord injury and the Physician indicates that a 90. Which of the following statements best describes
client is a Paraplegic. The family asks Nurse Marco severe depression?
what this means. What explanation should the nurse A. Anxiety and panic attack are characteristics of
give to the family? other condition
a. upper extremities are paralyzed B. Sonia wakes up early
b. lower extremities are paralyzed C. Sonia's depression is less in the morning
c. one side of the body is paralyzed but worsens toward the end of the day
d. both lower and upper extremities are paralyzed D. Delusion and hallucinations do not occur
85. Which clinical indicator does Nurse Marco identify
when assessing a client with hemiplegia? Situation: You are a nurse in Psychiatric Unit. The
a. paresis of both lower extremities use of therapeutic touch is an effective
b. paralysis of one side of the body intervention in caring for your patience.
c. paralysis of both lower extremities
d. paresis of upper and lower extremities 91. Which type of therapeutic touch is used when
you assess skin turgor of the patient during physical
Situation: Sonia is a 28 year old graduate school assessment?
student in a leading private university. She came A. Friendship C. Social
in a psychiatric unit due to sleeplessness but B. Love D. Functional
claiming she is still overactive. She revealed she 92. You gently guide a patient in going to her room.
has a history of depression when she was 18 This is the type of:
years old and was into drugs. Further evaluation A. Social B. Love
revealed Sonia is suffering from a Bipolar
C. Friendship D. Sexual
disorder, rapid cycling mood disorder
93. You put your arms around the shoulders of an
classification.
elderly patient. Which type of touch is this?
A. Love C. Social
86. Which of the following characterizes Rapid Cycling
Mood disorder? (not lke bipolar) B. Functional D. Friendship
A. Exaggerately energetic behavior 94. What type of touch is used when it involves tight
hugs and kisses between relatives?
B. Depressive episodes alternating with at least
one manic episode A. Love C. Social
C. Hypomanic episodes alternating with B. Friendship D. Functional
depressive episode of two years duration 95. Which of the following is NOT a type of
D. Two or more mood episodes of two therapeutic touch used by the nurse in providing care to
years duration psychiatric patients?
87. In your interview with Sonia, she said "I have special A. Social C. Sexual
power that's why I was sent by God to make this world B. Friendship D. Love
a better place." This is a manifestation of:
A. Paranoia Situation: Nurse Honeylet wants to improve in her
B. Delusion of persecution care for patient Mar, who is an alcoholic. She asks
C. Delusion of grandeur help from her head nurse Amanda.
D. Denial
88. You heard of Sonia telling another nurse, - there are 96. Honeylet goes to Mar's bedside to greet him.
people who wants to harm me because I have special Amanda corrects Honeylet of her greeting which is NOT
power." This is a manifestation of what behavior? appropriate to Mar
A. Mania C. Hallucination A. "Hi, Mar!„ so you got drunk last night"
B. Delusion of grandeur D. Delusion of B. "Hi, Mar!, I heard you enjoyed yourself last
persecution night"

TOPRANK REVIEW ACADEMY INC.


RECALLS 1 EXAM Page 53
C. "Hi, Mar!, I heard you had a drinking
spree last night"
D. "Hi, Mar!, How was your drinking affair
last night"

97. Mar turned his back away from Nurse Honeylet,


saying "It's none of your business, you ugly duckling."
The appropriate response of Nurse Honeylet would be:
A. "What you said hurt me, you alcoholic!"
B. "You beast, you are as ugly as I am"
C. "You really arc a drunkard"
D. "I don't think you mean what you have
just said. Do You?"

98. Amanda wished that Honeylet should have used


a/ an:
A. non-verbal communication
B. therapeutic communication
C. emphatic communication
D. casual communication

99. Amanda suggested that Honeylet should use


communication technique appropriate for the condition
of Mar such as:
A. Concluding C. questioning
B. Analyzing D. rephrasing

100. Honeylet should have used according to


Amanda, which kind of attitude?
A. motherly C. passive
B. friendly D. matter of fact

TOPRANK REVIEW ACADEMY INC.


RECALLS 1 EXAM Page 54

You might also like